30
1. Consider the following about Ibn Battuta’s account of India. 1. The highways were safe from robbery and other crimes. 2. Coconut fibre found in India was so strong that it was used for making ropes which pulled ships. 3. Indian textiles such as fine muslins were in great demand in Asian markets. 4. Horses and human runners were often used to dispatch goods required at short notice. Select the correct answer using the codes below. a) 1 and 3 only b) 2, 3 and 4 only c) 1 and 4 only d) 1, 2, 3 and 4 Solution: b) Justification: Statement 1: Ibn Battuta was a Moroccan traveler. When travelling in India, he was attacked by bands of robbers several times. In fact he preferred travelling in a caravan along with companions, but this did not deter highway robbers. While travelling from Multan to Delhi, his caravan was attacked and many of his fellow travellers lost their lives. Statement 2: His account of India says that “Coconut trees looked like date palms. It resembles a man’s head. Inside of it looks like a brain. Its fibre looks like human hair. Its fibre used for making rope which is used for pulling ships.” Statement 3: He also noted that the subcontinent was well integrated with inter-Asian networks of trade and commerce, with Indian manufactures being in great demand in both West Asia and Southeast Asia, fetching huge profits for artisans and merchants. Indian textiles, particularly cotton cloth, fine muslins, silks, brocade and satin, were in great demand. Statement 4: Further, almost all trade routes were well supplied with inns and guest houses. Ibn Battuta was also amazed by the efficiency of the postal system(by horse and human runners) which allowed merchants to not only send information and remit credit across long distances, but also to dispatch goods required at short notice. Q Source: Page 129: Themes in Indian History – II: 12th NCERT 2. India holds a strategic military advantage over Pakistan as the latter does not possess any Inter Continental Ballistic Missile (ICBM). The Indian ICBM missile series is named after? a) Agni b) Prithvi c) Akash d) BrahMos Solution: a) Learning: ICBMs are guided ballistic missile with a minimum range of 5,500 kilometres primarily designed for nuclear weapons delivery. Similarly, conventional, chemical, and biological weapons can also be delivered with varying effectiveness. ICBMs are differentiated by having greater range and speed than other ballistic missiles. • Agni-V: 2014, road and rail mobile ICBM has a range of 5,500- 8,000 km • Agni-VI (Under Development): is an ICBM with a range of 8,000- 12,000 km BrahMos is a short range missile, so D is wrong. Prithvi is only a ballistic missile. Other nations that have ICBMs are USA, France, China, Russia etc. Q Source: http://www.thehindubusinessline.com/news/india- successfully-testfires-indigenously-built-nuclear- capable-agnii-ballistic- missile-from-a-test-range-off-odisha-coast/article9373590.ece 3. China has launched world’s longest quantum communication line. Consider the following about it. Assertion (A): It is a hack-proof information transmission system. Reason (R): Quantum communication cannot be sent or captured from space to earth or vice versa. In the context of the above, which of these is correct? a) A is correct, and R is an appropriate explanation of A. b) A is correct, but R is not an appropriate explanation of A. c) A is correct, but R is incorrect. d) Both A and R are incorrect. Solution: c) Justification: Quantum communication uses subatomic particles to securely communicate between two points. These subatomic particles are mainly quantum entanglement of photons. It ensures that nobody taps into the line as a photon can be neither separated nor duplicated. Thus tapping the network will inevitably corrupt the signal. It has the ability to inform the two communicating users of the presence of any third party trying to eavesdrop. At the same time, the information being intercepted will self-destruct. So, A is correct. Why reason is invalid: Earlier in 2016, China had successfully launched the world’s first quantum satellite. The quantum satellite will be connected with the Beijing-Shanghai quantum communication line through the line’s station in Beijing. It will enable the space-to-Earth highly secure hack-proof quantum communication network. So, R is wrong.

stru file · Web view1. Consider the following about Ibn Battuta’s account of India. 1. The highways were safe from robbery and other crimes. 2. Coconut fibre found in India was

  • Upload
    lytu

  • View
    215

  • Download
    1

Embed Size (px)

Citation preview

Page 1: stru file · Web view1. Consider the following about Ibn Battuta’s account of India. 1. The highways were safe from robbery and other crimes. 2. Coconut fibre found in India was

1. Consider the following about Ibn Battuta’s account of India.1. The highways were safe from robbery and other crimes. 2. Coconut fibre found in India was so strong that it was used for making ropes which pulled ships. 3. Indian textiles such as fine muslins were in great demand in Asian markets. 4. Horses and human runners were often used to dispatch goodsrequired at short notice.Select the correct answer using the codes below.a) 1 and 3 only b) 2, 3 and 4 only c) 1 and 4 only d) 1, 2, 3 and 4Solution: b)Justification: Statement 1: Ibn Battuta was a Moroccan traveler. When travelling in India, he was attacked by bands of robbers several times. In fact he preferred travelling in a caravan along with companions, but this did not deter highway robbers. While travelling from Multan to Delhi, his caravan was attacked and many of his fellow travellers lost their lives.Statement 2: His account of India says that “Coconut trees looked like date palms. It resembles a man’s head. Inside of it looks like a brain. Its fibre looks like human hair. Its fibre used for making rope which is used for pulling ships.”Statement 3: He also noted that the subcontinent was well integrated with inter-Asian networks of trade and commerce, with Indian manufactures being in great demand in both West Asia and Southeast Asia, fetching huge profits for artisans and merchants. Indian textiles, particularly cotton cloth, fine muslins, silks, brocade and satin, were in great demand.Statement 4: Further, almost all trade routes were well supplied with inns and guest houses. Ibn Battuta was also amazed by the efficiency of the postal system(by horse and human runners) which allowed merchants to not only send information and remit credit across long distances, but also to dispatch goods required at short notice.Q Source: Page 129: Themes in Indian History – II: 12th NCERT2. India holds a strategic military advantage over Pakistan as thelatter does not possess any Inter Continental Ballistic Missile (ICBM). The Indian ICBM missile series is named after? a) Agni b) Prithvi c) Akash d) BrahMosSolution: a)Learning: ICBMs are guided ballistic missile with a minimum range of 5,500 kilometres primarily designed for nuclear weapons delivery. Similarly, conventional, chemical, and biological weapons can also be delivered with varying effectiveness.ICBMs are differentiated by having greater range and speed than other ballistic missiles.• Agni-V: 2014, road and rail mobile ICBM has a range of 5,500- 8,000 km• Agni-VI (Under Development): is an ICBM with a range of 8,000- 12,000 kmBrahMos is a short range missile, so D is wrong.Prithvi is only a ballistic missile.Other nations that have ICBMs are USA, France, China, Russia etc.Q Source: http://www.thehindubusinessline.com/news/india- successfully-testfires-indigenously-built-nuclear-capable-agnii-ballistic- missile-from-a-test-range-off-odisha-coast/article9373590.ece3. China has launched world’s longest quantum communication line.Consider the following about it.Assertion (A): It is a hack-proof information transmission system.Reason (R): Quantum communication cannot be sent or captured from space to earth or vice versa.In the context of the above, which of these is correct?a) A is correct, and R is an appropriate explanation of A. b) A is correct, but R is not an appropriate explanation of A. c) A is correct, but R is incorrect. d) Both A and R are incorrect.Solution: c)Justification: Quantum communication uses subatomic particles to securely communicate between two points. These subatomic particles are mainly quantum entanglement of photons. It ensures that nobody taps into the line as a photon can be neither separated nor duplicated.Thus tapping the network will inevitably corrupt the signal. It has the ability to inform the two communicating users of the presence of any third party trying to eavesdrop. At the same time, the information being intercepted will self-destruct. So, A is correct.Why reason is invalid: Earlier in 2016, China had successfully launched the world’s first quantum satellite.The quantum satellite will be connected with the Beijing-Shanghai quantum communication line through the line’s station in Beijing.It will enable the space-to-Earth highly secure hack-proof quantum communication network. So, R is wrong.Q Source: http://www.thehindu.com/news/international/China- launches-world%E2%80%99s-longest-quantum-communication- line/article16668990.ece4. Consider the following about Bundi School of Painting which hasbeen used to decorate many public places by the Rajasthan Government. 1. A characteristic feature of the school is the notable absence ofgreen colour which mirrors the ecology of Rajasthan. 2. The school is highly secular in nature and avoids depiction ofreligious themes.Which of the above is/are correct?a) 1 only b) 2 only c) Both 1 and 2 d) NoneSolution: d)Justification: Statement 1: This art form has been used to decorate the Kota Railway Station.• Bundi School of Painting is a Rajasthani style of Indian miniature painting that lasted from 17 the century towards the end of 19th century.• The Bundi School is characterized by a fondness for lush green vegetation, dramatic night skies and a distinctive way of depicting water with light swirls.• It basically existed in the princely state of Bundi and the neighbouring principality of Kotah.Statement 2: The Bundi Paintings emphasized on hunting, court scenes, procession, life of nobles, lovers, animals, birds and scenes from Lord Krishna’s life.The Bundi School has a close association with the Mughal style.Q Source: http://www.deccanherald.com/content/562735/railway- stations-turn-art-galleries.htmlhttp://indianexpress.com/article/lifestyle/art-and-culture/new-york- art-exhibition-mughal-royal-canvas-2960280/5. Metal salts presently used in firework displays in India include

Page 2: stru file · Web view1. Consider the following about Ibn Battuta’s account of India. 1. The highways were safe from robbery and other crimes. 2. Coconut fibre found in India was

1. Sodium nitrate 2. Potassium Chlorate 3. Strontium carbonateSelect the correct answer using the codes below.a) 1 and 2 only b) 2 and 3 only c) 1 and 3 only d) 1, 2 and 3Solution: c)Justification: Statement 2: It has been banned in India. It has an inherent property to become very reactive, especially when mixed with sulphur; the potassium chlorate-sulphur mixture becomes dangerously sensitive to friction and may spontaneously ignite.Hence, potassium chlorate is unsafe in fireworks.See http://www.thehindu.com/news/national/potassium-chlorate- beautiful-but-dangerous/article8468219.eceLearning: Inside each handmade firework are small packets filled with special chemicals, mainly metal salts and metal oxides, which react to produce an array of colors.• When heated, the atoms of each element in the mix absorb energy, causing its electrons to rearrange from their lowest energy state to a higher "excited" state.• Strontium carbonate (red fireworks), calcium chloride (orange fireworks), sodium nitrate (yellow fireworks), barium chloride (green fireworks) and copper chloride (blue fireworks).• Strontium compounds are also important for stabilizing fireworks mixtures. Titanium metal can be burned as powder or flakes to produce silver sparks. Zinc is a bluish white metal that is used to create smoke effects for fireworks.• Calcium helps deepen the colors of the other elements, and brings a silver color and a little bit of red to the fireworks.Q Source: Current Affairs + Improvisation: UPSC CDS 20136. Consider the following about National Legal Services Authority(NALSA). 1. It is a statutory body.2. It provides for free legal aid to the poor and weaker sections of the society. 3. It appoints judges of Gram Nyalayas in consultation with the District Judge. 4. It also organises Lok Adalats for amicable settlement ofdisputes.Select the correct answer using the codes below.a) 1, 2 and 4 only b) 2 and 4 only c) 1 and 3 only d) 1, 2, 3 and 4Solution: a)Justification: Statement 1: It has been constituted under the Legal Services Authorities Act, 1987Statement 2: Its aim is to ensure that opportunities for securing justice are not denied to any citizen by reasons of economic or other disabilities. NALSA also identifies specific categories of marginalised and excluded groups and formulates various schemes for implementation of legal service programmes.It provides services of free legal aid in civil and criminal matters for the poor and marginalised people who cannot afford the services of a lawyer in any court or tribunal.Statement 3: The Gram Nyayalayas are presided over by a Nyayadhikari, who has the same power; enjoys same salary and benefits of a Judicial Magistrate of First Class. Such Nyayadhikari are to be appointed by the State Government in consultation with the respective High Court.Statement 4: It also organises Lok Adalats for amicable settlement of disputes. It works in close coordination with various State Legal Services Authorities, District Legal Services Authorities and other agencies.Q Source: http://www.thehindu.com/todays-paper/tp- national/Khehar-made-NALSA-executive-chairman/article16655399.ece7. Food cans are generally coated with metals other than Zincbecause 1. Zinc can react with natural acids and bases in food materials. 2. Zinc promotes the growth of clostridium botilinium whichmakes food toxic.Which of the above is/are correct?a) 1 only b) 2 only c) Both 1 and 2 d) NoneSolution: a)Justification: Statement 1: Zinc is quite reactive. It can react with food items and make them unfit for consumption. Tin is used in place of zinc.Statement 2: The botulinum toxin is produced in the food during anaerobic growth, not due to presence of Zinc. We have covered the toxin and related issues in a previous test.Q Source: Improvisation: UPSC CDS 20138. Consider the following associated with Buddha’s life.1. Alara Kalma: Buddha’s teacher 2. Kanthaka: Place where Buddha first stopped after taking Sanyasa 3. Channa: Charioteer of Gautam SiddharthaSelect the correct matches using the codes below.a) 1 and 2 only b) 2 and 3 only c) 1 and 3 only d) 1, 2 and 3Solution: c)Justification: Statement 1: After Gautama became an ascetic, he went to Alara Kalama, who was a teacher that taught a kind of early samkhya at Vessali. Alara taught Gautama Buddha meditation techniques.Statement 2: Kanthaka was the royal horse of Gautama Buddha. It was a favourite horse of the prince. Legend says that Siddhartha and Kanthaka were born on the same dayStatement 3: Channa was the servant who served as the charioteer pulled by the horse Kanthaka, when Siddhartha saw the Four sights (old age, sickness, a corpse and an ascetic) whilst meeting his subjects in the Sakya capital Kapilavastu, which prompted his decision to renounce the world.Q Source: Improvisation: UPSC CDS 20139. RBI operates a Strategic Debt Restructuring (SDR) scheme. Whatdoes Debt Restructuring mean? 1. Writing-off debts to improve bank’s account sheet 2. Converting the debt to equity and trying to recover loan by employing sound managerial techniques in defaulter firmsWhich of the above is/are correct?a) 1 only b) 2 only c) Both 1 and 2 d) NoneSolution: b)Justification: Statement 1: Writing off debt means that asset has turned from a debt to a bad asset, with little hope of recovery. Banks do this in order to improve their current balance sheet. Writing off loans does not mean waiving them.Statement 2: Once an asset is recognised as a non-performing asset (NPA), banks must decide what to do with it. They have several options.

Page 3: stru file · Web view1. Consider the following about Ibn Battuta’s account of India. 1. The highways were safe from robbery and other crimes. 2. Coconut fibre found in India was

• One, they can try to seize the assets pledged by the borrower and sell these. This typically involves large losses on loans as the assets have to be sold at steep discounts to their book value.• Two, under the RBI’s Strategic Debt Restructuring (SDR) scheme, they can convert their loans into equity, acquire a majority stake in the firm, dislodge the promoters or management and bring in new promoters and management.• While this happens in advanced economies all the time, the SDR scheme has not taken off in India.• Indian banks do not have experience in running businesses till such time as new promoters are found. Nor do they have experience in locating promoters and management who can take over the stressed assets.Q Source: http://indianexpress.com/article/business/economy/bad- loans-write-off-sbi-vijay-mallya-arun-jaitley-kingfisher-4378675/10. Photochemical smog has recently become a major problem in cities like New Delhi. Which of the following is/are crucial for the formation of Photochemical smog in the atmosphere? a) Sunlight b) Volatile organic compounds c) Tropospheric ozone d) All of the aboveSolution: d)Learning: Photochemical smog is the chemical reaction of sunlight, nitrogen oxides and volatile organic compounds in the atmosphere, which leaves airborne particles and ground-level ozone. It may include the following:• Aldehydes• Nitrogen oxides, particularly nitric oxide and nitrogen dioxide• Peroxyacyl nitrates• Tropospheric ozone• Volatile organic compoundsAll of these harsh chemicals are usually highly reactive and oxidizing. Photochemical smog is therefore considered to be a problem of modern industrialization.It is present in all modern cities, but it is more common in cities with sunny, warm, dry climates and a large number of motor vehicles like in Delhi.Q Source: Improvisation: Page 137: 12th NCERT: India, People and Economy11. Excessive generation of fly ash can lead to which of these harmfulenvironmental effects? 1. It can result in leaching of toxic heavy metals in ground water. 2. It contains crystalline silica which can cause silicosis.Which of the above is/are correct?a) 1 only b) 2 only c) Both 1 and 2 d) NoneSolution: c)Justification: Statement 1: When pulverized coal (bituminous/lignite is burnt in the boiler of a thermal power station, a part of ash falls down at the bottom of the boiler and is known as bottom ash.• Fly ash contains trace concentrations of heavy metals and other substances that are known to be detrimental to health in sufficient quantities.• Potentially toxic trace elements in coal include arsenic, beryllium, cadmium, barium, chromium, copper, lead, mercury, molybdenum, nickel, radium, selenium, thorium, uranium, vanadium, and zinc.Statement 2: A fraction of the mass of coal burned consists of unburnable mineral that becomes ash, so the concentration of most trace elements in coal ash can be very high as compared to the coal.Fly ash contains crystalline silica which is known to cause lung disease, in particular silicosis.Learning: Maharashtra became the first state in the country to adopt the Fly Ash Utilization Policy.• The policy seeks 100% use of fly ash generated from thermal power plants and biogas plants for construction activities. Facilitates use of fly ash to make bricks, blocks, tiles, wall panels, cement and other construction materials.• It will help in environment protection and save soil excavation. It also will make available raw material for construction at low cost to help ‘Housing for All’ projects.Q Source: http://www.business-standard.com/article/pti- stories/maha-becomes-first-state-to-adopt-fly-ash-utilisation-policy- 116111500919_1.html12. Consider the following statements.Assertion (A): All major ports in India are presently being administered or managed by either the Central government or State governments.Reason (R): As per the constitution, maritime transport can be administered by both the Central and the State governments.In the context of the above, which of these is correct?a) A is correct, and R is an appropriate explanation of A. b) A is correct, but R is not an appropriate explanation of A. c) A is incorrect, but R is correct.d) Both A and R are incorrect.Solution: c)Justification: Major ports are generally owned and managed by the Central government; minor ports by the State government. So, A is anyways incorrect.All major ports, except Ennore Port are government administered. This also shows that A is incorrect.Private sector participation in ports has increased substantially. Many shipping companies are privately owned.Entry 31 and 32 of the 7th Schedule (Concurrent List) read:• Ports other than those declared by or under law made by Parliament or existing law to be major ports.• Shipping and navigation on inland waterways as regards mechanically propelled vessels, and the rule of the road on such waterways, and the carriage of passengers and goods on inland waterways subject to the provisions of List I with respect to national waterways.• The above can be managed by both Central and State governments.Q Source: Improvisation: Page 130: 12th NCERT: India, People and Economy13. Consider the following with reference to the forest Society andtribes in the Mughal Period. 1. Forest tribes often supplied elephants to the state as a tribute. 2. Forests provided gum lac which was a major export item overseas from India. 3. The state banned private commercial agriculture in forestregions to preserve their pristine ecology.Select the correct answer using the codes below.a) 1 and 2 onlyb) 2 and 3 only c) 3 only d) 1, 2 and 3Solution: a)Justification: Statement 1: For the state, the forest was a place of rebels and troublemakers. Babur says that jungles provided a good defence “behind which the people of the pargana become stubbornly rebellious and pay no taxes”.

Page 4: stru file · Web view1. Consider the following about Ibn Battuta’s account of India. 1. The highways were safe from robbery and other crimes. 2. Coconut fibre found in India was

The state required elephants for the army. Elephants were captured from forest and sold. So the peshkash (tribute) levied from forest people often included a supply of elephants.Statement 2 and 3: The spread of commercial agriculture was an important external factor that impinged on the lives of tribals. So, 3 is wrong.Forest products –like honey, beeswax and gum lac – were in great demand. Some, such as gum lac, became major items of overseas export from India in the seventeenth century.Q Source: Page 208: Themes in Indian History – II: 12th NCERT14. Indian Remote Sensing (IRS) satellites can be used for which ofthese applications? 1. Urban planning and forest surveys 2. Mineral Prospecting 3. Telecommunications and tele-networking 4. Locating groundwater resourcesSelect the correct answer using the codes below.a) 1 and 4 only b) 1, 2 and 4 only c) 2 and 3 only d) 1, 2, 3 and 4Solution: b)Justification: Remote sensing is the science of obtaining information about objects or areas from a distance, typically from aircraft or satellites.• Remote sensors collect data by detecting the energy that is reflected from Earth. These sensors can be on satellites or mounted on aircraft.• All these are placed in polar Sun-synchronous orbit. Varieties of instruments have been flown onboard these satellites to provide necessary data in a diversified spatial, spectral and temporal resolutions to cater to different user requirements in the country and for global usage.Remote sensing has a wide range of applications in many different fields:• Coastal applications: Monitor shoreline changes, track sediment transport, and map coastal features. Data can be used for coastal mapping and erosion prevention.• Ocean applications: Monitor ocean circulation and current systems, measure ocean temperature and wave heights, and track sea ice. Data can be used to better understand the oceans and how to best manage ocean resources.• Hazard assessment: Track hurricanes, earthquakes, erosion, and flooding. Data can be used to assess the impacts of a natural disaster and create preparedness strategies to be used before and after a hazardous event.• Natural resource management: Monitor land use, map wetlands, and chart wildlife habitats. Data can be used to minimize the damage that urban growth has on the environment and help decide how to best protect natural resources.Q Source: Improvisation: Page 123: 12th NCERT: India, People and Economy15. The term ‘Exoplanet’ often seen in news is used fora) A planet that does not orbit the Sun and instead orbits a different star b) A dwarf planet that is a part of our solar system c) A large asteroid that supports near life conditions d) A brown stellar remnant that orbits our solar systemSolution: a)Learning: Scientists have discovered a new ‘super Earth’ type exoplanet named as GJ 536b orbiting a very bright star near to our Sun.The planet has a very short orbital period (nearly 9 days only) which would come handy in conducting future studies of biological activity.An Exoplanet is a planet that does not orbit the Sun and instead orbits a different star, stellar remnant, or brown dwarf. It is also termed as extrasolar planet.Super-earths are those extrasolar planets which are larger than the Earth. However they have mass substantially below the solar system’s giant planets namely Neptune and Uranus.Q Source: https://www.sciencedaily.com/releases/2016/11/161116102103.htm16. Valuable minerals in India are more likely to be associated witha) Blue quartz phenocrysts b) Metamorphic and igneous rocks c) Basaltic rocks of andesite and rhyolite type d) Sedimentary and Haedan age rocksSolution: b)Learning: You should not be intimidated by strange sounding options. You can get the answer by common sense.• Quartz and Basalt both can be associated with sedimentary rocks, even though Quartz may occur in felsic igneous rocks or granite. Option D is also about sedimentary rocks, so all can be eliminated except B.• Bulk of the valuable minerals in India are products of pre- Palaeozoic age spanning from nearly 540-250 million years ago. Hadean age refers to the one right after formation of earth. So, D can be anyways eliminated.• These minerals are mostly associated with metamorphic and igneous rocks of the peninsular India, and not sedimentary rocks.• The vast alluvial plain tract of north India is devoid of minerals of economic use.Q Source: Page 72: 12th NCERT: India, People and Economy17. Which of the following countries are members of the EuropeanOrganisation for Nuclear Research (CERN)? 1. India 2. Pakistan 3. Russia 4. United KingdomSelect the correct answer using the codes below.a) 1 and 4 only b) 1, 2 and 3 only c) 4 only d) None of the aboveSolution: c)Justification: India recently joined CERN as associate member (not full member).CERN has 22 member states (including UK), four associate member states (including India and Pakistan) and three International Organisations have observer status. Russia has observer status.India had ‘observer’ status till September 2016, when the CERN Council adopted a resolution upgrading its position.Member states have special duties and privileges. They make a contribution to the capital and operating costs of CERN’s programmes, and are represented in the council, responsible for all important decisions about the organization and its activities.Learning: CERN as an organisation is world’s largest nuclear and particle physics laboratory. It is situated in North West suburbs of Geneva on France-Swiss Border. It was established in 1954.CERN operates the Large Hadron Collider (LHC) which is the world’s largest and most powerful particle accelerator. It is associated with the discovery of the Higgs Boson which is popularly known as the God particle.Q Source: http://www.livemint.com/Science/xRZDQSCQ71qOUd8gxxOpbI/India- becomes-associate-member-of-CERN.html18. The ‘Gandhi-Irwin Pact’ included which of the followingcommitments by both the parties? 1. Civil disobedience movement will be called off. 2. British would reduce land rent by half in areas of mahalwari settlements. 3. Income and corporation taxes would be reduced. 4. Military expenditure on foreign lands would not be financedfrom Indian exchequer.Select the correct answer using the codes below.

Page 5: stru file · Web view1. Consider the following about Ibn Battuta’s account of India. 1. The highways were safe from robbery and other crimes. 2. Coconut fibre found in India was

a) 1 onlyb) 2 and 3 only c) 1 and 2 only d) 3 and 4 onlySolution: a)Justification: Salient features of this act were as following:• The Congress would participate in the Round Table Conference.• The Congress would discontinue the Civil Disobedience Movement.• The Government would withdraw all ordinances issued to curb the Congress.• The Government would withdraw all prosecutions relating to offenses other than violent one.• The Government would release all persons undergoing sentences of imprisonment for their activities in the civil disobedience movement.• Manufacture of salt by Indians will be allowed.The pact was criticised by radical nationalists, for Gandhiji was unable to obtain from the Viceroy a commitment to political independence for Indians; he could obtain merely an assurance of talks towards that possible end.Q Source: Page 360: Themes in Indian History – II: 12th NCERT19. The functions of the Atomic Energy Commission is/are1. To undertake prospecting and extraction of atomic minerals in India 2. To organise research in atomic energy and train scientists in thecountryWhich of the above is/are correct?a) 1 only b) 2 only c) Both 1 and 2 d) NoneSolution: c)Justification: The Atomic Energy Commission was set up in 1948 to look after atomic energy activities in the country. The functions of the Atomic Energy Commission are:• to organise research in atomic scientists in the country;• to train, atomic scientists in the country;• to promote nuclear research in commission's own laboratories as well as in India;• to undertake prospecting of atomic minerals in India and to extract such minerals for use on industrial scale.Learning: The Secretary to the Government of India in the Department of Atomic Energy is ex-officio Chairman of the Commission. The other Members of the AEC are appointed for each calendar year on the recommendation of the Chairman, AEC and after approval by the Prime Minster.Q Source: Improvisation: Page 82: Chapter 7: 12th NCERT: India, People and Economy20. The Dandi March was launched against1. State monopoly on manufacture and sale of salt 2. Exorbitantly high salt taxWhich of the above is/are correct?a) 1 onlyb) 2 only c) Both 1 and 2 d) NoneSolution: c)Justification: Gandhi wrote, “The salt monopoly is thus a fourfold curse. It deprives the people of a valuable easy village industry, involves wanton destruction of property that nature produces in abundance, the destruction itself means more national expenditure, and fourthly, to crown this folly, an unheard-of tax of more than 1,000 per cent is exacted from a starving people.”He further explained, “In order to prevent the use of salt that has not paid the tax which is at times even fourteen times its value, the Government destroys the salt it cannot sell profitably. Thus it taxes the nation’s vital necessity; it prevents the public from manufacturing it and destroys what nature manufactures without effort.”The Dandi march was launched against this background and became successful in allowing people the right to manufacture salt.Q Source: Page 356-357: Themes in Indian History – II: 12th NCERT21. In India, which of these geological formations hosts the mostsignificant ground water reservoirs for large scale and extensive development? a) Unconsolidated Formations b) Semi-Consolidated Formations c) Fissured Formations d) Consolidated FormationsSolution: a)Learning: The ground water behaviour in the Indian sub-continent is highly complicated due to the occurrence of diversified geological formations with considerable difference in tectonic and lithological framework.Broadly two groups of rock formations have been identified depending on characteristically different hydraulics of ground water, Viz. Porous Formations and Fissured Formations.Porous Formations have been further subdivided into Unconsolidated and Semi – consolidated formations.Unconsolidated Formations• The areas covered by alluvial sediments of river basins, coastal and deltaic tracts constitute the unconsolidated formations. These are by far the most significant ground water reservoirs for large scale and extensive development.• The hydro-geological environment and ground water regime in the Indo-Ganga-Brahmaputra basin indicate the existence of potential aquifers having enormous fresh ground water reserve.Semi-Consolidated Formations• The semi-consolidated formations normally occur in narrow valleys or structurally faulted basins. The Gondwanas, Lathis, Tipams, Cuddalore sandstones and their equivalents are the most extensive productive aquifers in this category.Fissured Formations (Consolidated Formations)• The consolidated formations occupy almost two-thirds of the country. These formations, except vesicular volcanic rocks have negligible primary porosity and thus low groundwater extraction potential.Q Source: Improvisation: Page 62: 12th NCERT: India, People and Economy22. The Dasara Dibba is one of the most impressive structures inHampi. It was used for which of the following purposes by the King? 1. Watching army marches and war games 2. Performing sacred YagnasWhich of the above is/are correct?a) 1 only b) 2 only c) Both 1 and 2 d) NoneSolution: a)

Page 6: stru file · Web view1. Consider the following about Ibn Battuta’s account of India. 1. The highways were safe from robbery and other crimes. 2. Coconut fibre found in India was

Justification: Statement 1: The Dasara Dibba attracts attention as one of the tallest structures in the Royal Enclosure of Hampi. The stone platform has a height of about 12 metres.• The grand platform was used as a stage by the kings to watch the celebration of the nine-day long splendid Mahanavami festival, also known as the Dasara festival. Due to this reason the platform came to be known as Mahanavami Dibba.• The ornamental platform is a high structure on which the king used to sit and watch the army march-past, the war games and the royal procession that were held during the festival.• Dasara was the annual state festival and it was celebrated in style by the Vijayanagara kings.Q Source: Page 181: Themes in Indian History – II: 12th NCERT23. Aravallis in Rajasthan and Srisailam forests in AndhraPradesh are known for hosting a) Uranium mines b) Gold mines c) Diamond mines d) Platinum minesSolution: a)Learning: Uranium deposits occur in the Dharwar rocks. Geographically, uranium ores are known to occur in several locations along the Singbhum Copper belt.Recently uranium reserves were discovered in pockets of Mahbubnagar, Karimnagar and Nalgonda districts in Telangana and Guntur and Kadapa districts in Andhra Pradesh.Recently reserves were discovered in Srisailam forests.http://timesofindia.indiatimes.com/india/Major-uranium-reserves- found-in-Srisailam-forests/articleshow/48181983.cmsQ Source: Improvisation: Page 80: 12th NCERT: India, People and Economy24. In a ‘Paradox of Thrift’ situation1. Individuals end up saving less in a bid to save more 2. Governments default on debt in the long-run despiteconsiderable investment in human and physical capital 3. Investment does not pick up despite lower interest rates andavailable opportunitiesSelect the correct answer using the codes below.a) 1 only b) 1 and 2 only c) 2 and 3 only d) 1 and 3 onlySolution: a)Justification: As per the paradox of thrift, individuals try to save more during an economic recession, which essentially leads to a fall in consumption. As consumption falls, aggregate demand reduces and hence there is a fall in economic growth.As growth is lowered, the income of people reduces. Since our savings and consumption depend on our income, with lower incomes we end up saving less; hence, the paradox.Such a situation is harmful for everybody as investments give lower returns than normal.Q Source: http://economictimes.indiatimes.com/definition/paradox- of-thrifthttp://www.economist.com/blogs/buttonwood/2015/12/other-paradox- thrift25. Consider the following about Maize crop production in India.1. Maize is cultivated throughout the year in all states of the country. 2. It is the only crop for which no “cross hybrids” have beenintroduced for field trials or commercial production.Which of the above is/are correct?a) 1 only b) 2 only c) Both 1 and 2 d) NoneSolution: a)Justification: Statement 1: The maize is cultivated throughout the year in all states of the country. Maize can be grown in all seasons viz; Kharif (monsoon), post monsoon, Rabi (winter) and spring.• The predominant maize growing states that contribute more than 80 % of the total maize production are Andhra Pradesh (20.9 %), Karnataka (16.5 %), Rajasthan (9.9 %) etc.• Maize can be grown successfully in variety of soils ranging from loamy sand to clay loam. However, soils with good organic matter content having high water holding capacity with neutral pH are considered good for higher productivity.Statement 2: Globally, maize is known as queen of cereals because it has the highest genetic yield potential among the cereals. As per a government document, “With the increasing trends of maize production, the projected demand of maize by the end of XIth five year plan (2011- 12) will be achieved through improved maize production technologies focused on ‘Single Cross Hybrids’.” This means statement 2 is clearly incorrect.You can refer to this link for more information http://farmer.gov.in/(S(2225rkhyjpn5zeviigrhwoec))/imagedefault/pest anddiseasescrops/normalmaizeproductiontechnologies.pdfQ Source: Page 47: 12th NCERT: India, People and Economy26. Higher education finance agency (HEFA), recently in news,will finance 1. Creation of high quality infrastructure in premier educational institutions 2. Scholarships of students from economically weaker sectionsaspiring for higher education from government institutionsWhich of the above is/are correct?a) 1 onlyb) 2 only c) Both 1 and 2 d) NoneSolution:Justification: The Union Cabinet has approved the creation of the Higher Education Financing Agency (HEFA) to give a major push for creation of high quality infrastructure in premier educational institutions.• HEFA will have around Rs 1100 crore of equity that will be used to raise funds from the markets for lending to educational institutions.• It would leverage equity to fund projects for infrastructure and development of world class Labs in IITs/IIMs/NITs and such other institutions.• It will be jointly promoted by the identified Promoter and the Ministry of Human Resource Development.• It would be formed as a SPV within a PSU Bank/ Government- owned-NBFC (Promoter).• It would also mobilise CSR funds from PSUs/Corporates, which would in turn be released for promoting research and innovation in these institutions on grant basis.• The principal portion of the loan will be repaid through the ‘internal accruals’ (earned through the fee receipts, research earnings etc) of the institutions. The Government would service the interest portion through the regular Plan assistance.Q Source: http://indianexpress.com/article/education/cabinet- approves-setting-up-of-higher-education-financing-agency-3027148/

Page 7: stru file · Web view1. Consider the following about Ibn Battuta’s account of India. 1. The highways were safe from robbery and other crimes. 2. Coconut fibre found in India was

27. The categorization of Indian cities in different tiers (I, II, III)is done on the basis of a) Size of manufacturing sectorb) Standard of living c) Population size d) Distance from capital citiesSolution: c)Learning: There are many ways to categorize cities in India.The Reserve Bank of India (RBI) classifies centres into six tiers based on population.• Tier-1- 100,000 and above• Tier-2 - 50,000 to 99,999• Tier-3 - 20,000 to 49,999• Tier-4 - 10,000 to 19,999• Tier-5 - 5,000 to 9,999• Tier-6 - less than 5000 (rural)The Classification of Indian cities also comprises a ranking system used by the Government of India to allocate House Rent Allowance (HRA) to public servants employed in different cities in the country – X (metropolitans), Y and Z.A Metropolitan area has a population of ten lakhs (one million) or more, and comprises one or more districts comprising two or more Municipalities or Panchayats or other contiguous area, specified by the Governor by public notification.Q Source: Improvisation: Page 36-37: 12th NCERT: India, People and Economy28. The unemployment that results due to weak seasonaleconomic demand is generally termed as a) Frictional unemployment b) Natural rate of unemployment c) Cyclical unemployment d) Structural unemploymentSolution: c)Learning: Frictional unemployment is the unemployment which exists in any economy due to people being in the process of moving from one job to another.Structural unemployment is a form of unemployment caused by a mismatch between the skills that workers in the economy can offer, and the skills demanded of workers by employers (also known as the skills gap).Natural rate of unemployment is the sum of Frictional and structural unemployment.Cyclical unemployment is a factor of overall unemployment that relates to the cyclical trends in growth and production that occur within the business cycle. When business cycles are at their peak, cyclical unemployment will be low because total economic output is being maximized.Q Source: Frequently in news29. The Lahore Session of the Indian National Congress (INC)was significant for which of the following reasons? 1. Election of Gandhiji as INC President 2. Proclamation of commitment to Purna SwarajWhich of the above is/are correct?a) 1 only b) 2 only c) Both 1 and 2 d) NoneSolution: b)Justification: Statement 1: Gandhiji never served as INC President, and rescued himself from formal political responsibilities. It was Pandit Nehru who was elected. His election signified the passing of the baton of leadership INC to the younger generation.Statement 2: “Purna Swaraj”, or complete independence declaration was proclaimed for the first time. Now the pace of politics picked up once more. On 26 January 1930, “Independence Day” was observed, with the national flag being hoisted in different venues.Q Source: Page 355: Themes in Indian History – III: 12th NCERT30. Which of these types of goods best qualifies as a public good?a) Rival and exclusive goods b) Non-Rival and exclusive goods c) Rival and Non-exclusive goods d) Non-Rival and Non-exclusive goodsSolution: d)Justification: A rival good is a good whose consumption by one consumer prevents simultaneous consumption by other consumers, for e.g. a berth on train. But, not all public goods are rival, for e.g. defense, police security, public parks etc.Excludable goods are those for which one can at low cost prevent those who have not paid for the good from consuming it. You can require people to pay for a stamp before you deliver mail or pay for a ticket before they board a train; you cannot cheaply or easily prevent people from entering a park or from listening to a radio station.So, any good that has both characteristics can be only provided by the government. It will be economically unviable for the private sector to provide such goods.Q Source: Often in news31. Gandhiji listed seven rules as “essential for every Satyagrahi inIndia”. These rules included that the Satyagrahi must 1. have living faith in God 2. have faith in the inherent goodness of human nature 3. be a habitual khadi wearer and spinnerSelect the correct answer using the codes below.a) 1 only b) 2 and 3 only c) 3 only d) 1, 2 and 3Solution: d)Justification: He believed that these rules should be taught to everyone. Apart from the rules mentioned above, he said that a Satyagrahi• must believe in truth and non-violence and have faith in the inherent goodness of human nature which he expects to evoke by suffering in the satyagraha effort• must be leading a chaste life, and be willing to die or lose all his possessions• must abstain from alcohol and other intoxicants• must willingly carry out all the rules of discipline that are issued• must obey the jail rules unless they are specially devised to hurt his self-respectQ Source: Improvisation: Page 348: Themes in Indian History – II: 12th NCERT32. Sufi Silsilas became a major tradition in Medieval India. Theterm silsila implies a) A family of guest houses for spiritual debates b) A spiritual genealogy linking the master to the disciples c) A method of chanting leading to communion with God d) A distinct way of Sufi poetry that later developed as a matureClassical styleSolution: b)

Page 8: stru file · Web view1. Consider the following about Ibn Battuta’s account of India. 1. The highways were safe from robbery and other crimes. 2. Coconut fibre found in India was

Learning: Institutionally, the sufis began to organise communities around the hospice or khanqah (Persian) controlled by a teaching master known as shaikh (in Arabic), pir or murshid (in Persian).Sufi silsilas began to crystallise in different parts of the Islamic world around the twelfth century. The word silsila literally means a chain, signifying a continuous link between master and disciple, stretching as an unbroken spiritual genealogy to the Prophet.Q Source: Page 153: Themes in Indian History – II: 12th NCERT33. Prepaid Payment Instruments (PPIs) in India are regulatedby a) Securities and Exchange Board of India (SEBI) b) Payment Settlement Authority of India (PSAI) c) Reserve Bank of India d) Insurance Regulatory and Development Authority (IRDA)Solution: c)Learning: PPIs are methods that facilitate purchase of goods and services against the value stored on such instruments.These prepaid instruments can be issued as online wallets (e.g. paytm), mobile accounts, mobile wallets, smart cards, magnetic stripe cards, internet accounts, paper vouchers and any such instruments used to access the prepaid amount.Recently, Reserve bank of India (RBI) has increased the per month limit of Prepaid Payment Instruments (PPIs) to Rs. 20,000 from Rs. 10,000 and to Rs. 50,000 for merchant bank in view of demonetization measures.Q Source: Recent Demonetization of currency notes34. Yavanas that are mentioned in detail in Sangam literaturerefer to a) Some Greek kingdoms b) Elaborate religious arrangements made by Kings c) Horse chariots that were used for Ashvamedha sacrifices d) Foreign conquests that lead to local cultural assimilationSolution: a)Learning: In some Sanskrit sources, the usage of the words "Yona", "Yauna", "Yonaka", "Yavana" or "Javana" etc. appears repeatedly, and particularly in relation to the Greek kingdoms which neighbored or sometimes occupied the Punjab region over a period of several centuries from the 4th century BCE to the first century CE.Examples are the Seleucid Empire, the Greco-Bactrian Kingdom and the Indo-Greek Kingdom.The Yavanas are mentioned in detail in Sangam literature epics such as Pattinappalai, describing their brisk trade with the Early Cholas in the Sangam period.Q Source: Page 174: Themes in Indian History – II: 12th NCERT35. Consider the following statements with reference todevelopments post-1991 economic reforms. 1. Income inequality has increased in India as compared to the status in 1991. 2. Human Development Index (HDI) ranking of India has fallencontinuously since 1991.Which of the above is/are correct?a) 1 only b) 2 only c) Both 1 and 2 d) NoneSolution: a)Justification: Statement 1: Inequality in earnings has doubled in India over the last two decades, making it the worst performer on this count of all emerging economies. The top 10% of wage earners now make 12 times more than the bottom 10%, up from a ratio of six in the 1990s. So, 1 is correct.Statement 2: India continued to rank low in the Human Development Index (HDI), but climbed five notches to the 130th rank in the latest UNDP report (2015) on account of rise in life expectancy and per capita income. India's 2014 HDI of 0.609 is below the average of 0.630 for countries in the medium human development group.So, even if you do not the trends post-1991, it could have been answered with the help of recently available information. 2 is incorrect.Q Source: Credit Suisse Report 2016 : Income inequality in India36. The Compact Muon Solenoid (CMS) and ATLAS experiment are sometimes seen in news. They are associated with research on 1. Dark matter 2. Higgs Boson 3. Extra DimensionsSelect the correct answer using the codes below.a) 1 and 2 only b) 3 only c) 1 and 3 only d) 1, 2 and 3Solution: d)Justification: Statement 1 and 2: ATLAS detector is probing for fundamental particles.ATLAS is one of two general-purpose detectors at the Large Hadron Collider (LHC). It investigates a wide range of physics, from the search for the Higgs boson to extra dimensions and particles that could make up dark matter.Although it has the same scientific goals as the CMS experiment, it uses different technical solutions and a different magnet-system design.The CMS experiment is one of the largest international scientific collaborations in history.Statement 3: Extra Dimensions have been explained in very simple terms herehttps://home.cern/about/physics/extra-dimensions-gravitons-and-tiny- black-holesQ Source: http://www.livemint.com/Science/xRZDQSCQ71qOUd8gxxOpbI/India- becomes-associate-member-of-CERN.html37. Which of these personalities associated with the Indiangovernment for sometime have also served as the Chief Economist of either World Bank or IMF? 1. Kaushik Basu 2. Raghuram G Rajan 3. Arvind Subramanium 4. IG Patel 5. Shankar AcharyaSelect the correct answer using the codes below.a) 1 and 2 only b) 2, 3 and 4 only c) 1, 3 and 5 only d) 1, 2, 4 and 5 onlySolution: a)Justification: You can go through the following lists https://en.wikipedia.org/wiki/List_of_IMF_Economic_Counsellorshttps://en.wikipedia.org/wiki/World_Bank_Chief_Economisthttps://en.wikipedia.org/wiki/Chief_Economic_Adviser_to_the_Gover nment_of_IndiaRecently Paul Romer, an eminent economist was appointed as the Chief Economist of World Bank.Q Source: http://www.worldbank.org/en/news/press- release/2016/07/18/world-bank-group-president-appoints-paul-romer- as-chief-economist + Improvisation: UPSC CDS 201338. ‘Project Insight’ will be launched by the government toa) Understand the complex climate change patters caused due to melting of Himalayan Glacier b) Gain Insights into pending Genetically Modified (GM) crop field trials c) Monitor high value transactions to curb black money d) Decode the communication pattern of ‘hate messages’ on socialmedia websites to tackle terrorism and communalism

Page 9: stru file · Web view1. Consider the following about Ibn Battuta’s account of India. 1. The highways were safe from robbery and other crimes. 2. Coconut fibre found in India was

Solution: c)Learning: The Income Tax department is planning to implement the first phase of ‘Project Insight’ from May 2017 to monitor high value transactions, with a view to curbing the circulation of black money.This project has been initiated for collection, collation and processing of such information for effective risk management with a view to widening and deepening tax base.It will use data mining, big data and analytics to scoop out tax evaders from social media platforms like Facebook, Twitter and Instagram. The Permanent Account Number (PAN) will be the unique identifier is used by the Income Tax department to link and analyse various transactions relating to the tax payers.Q Source: http://indianexpress.com/article/business/business- others/cbdt-will-use-data-from-project-insight-to-widen-tax-base-rani- singh-nair-3067432/http://indianexpress.com/article/business/business-others/cbdt-will- use-data-from-project-insight-to-widen-tax-base-rani-singh-nair- 3067432/39. Which of the following correctly differentiates between thecommitments made under the Kyoto Protocol (1997) and Paris Agreement (2016)? 1. Kyoto Protocol was legally binding, Paris Agreement is not. 2. Paris Agreement sets specific emission reduction targets onparty nations, whereas Kyoto protocol relied on voluntary commitments.Which of the above is/are correct?a) 1 only b) 2 only c) Both 1 and 2 d) NoneSolution: a)Justification: Statement 1: Unlike the Kyoto Protocol, Paris Agreement is not legally-binding and it does not assign any emission cut targets on any country.Statement 2: Under Paris Agreement, every country decides for itself the actions that it wants to take to fight climate change.These are called Intended Nationally Determined Contributions (INDCs).Learning: Doha Amendments to the Kyoto Protocol was made at Doha climate conference in 2012. It was made for extending the obligation of the developed countries under Kyoto Protocol to make targeted cuts in their greenhouse gas (GHGs) emissions by 2020. The Kyoto Protocol adopted in 1997 had initially assigned GHGs emission cut targets to the developed countries only till 2012.Q Source: http://www.un.org/sustainabledevelopment/blog/2016/11/countries-at- cop22-pledge-to-press-ahead-with-implementation-of-paris-agreement/http://www.thehindu.com/news/marrakech-action-proclamation- sends-out-strong-signal-on-climate/article9361369.ece40. Consider the following matches of important officials in theMughal Empire and their responsibilities. 1. Fotedar : Treasurer 2. Diwan-i-Rasalat : Army Commander 3. Karkuns : Engineering structures 4. Diwan-i-Insha : Minister for CommunicationsSelect the correct answer using the codes below.a) 1 and 4 only b) 2 and 3 only c) 1, 3 and 4 only d) 1, 2 and 3 onlySolution: a)Justification: Statement 1 and 3: The treasurer was Fotehdar and the entire amount collected was kept in his custody. He also maintained account of the income and expenditure of the paragana. Besies Fotedar, Karkuns handled matters related to accounts of the Paraganas.Statement 2 and 4: The central government of Sher Shah Suri consisted of several departments. The king was assisted by four important ministers:• Diwan –i- Wizarat – (Wazir) – in charge of Revenue and Finance.• Diwan-i-Ariz – in charge of Army.• Diwan-i-Rasalat- Foreign Minister.• Diwan-i-Insha- Minister for Communications.Q Source: Improvisation: Page 214: Themes in Indian History – II: 12th NCERT41. Which of the following always remains constant during the short-term production process of a firm? a) Marginal Cost b) Average Fixed Cost c) Marginal Revenue Productivity of Capital d) None of the aboveSolution: d)Justification: Option A: Marginal cost is the cost of producing an extra unit. It does not remain constant. It may decrease if the returns to scale are increasing; increase if returns to scale decrease; or remain constant if constant returns to scale prevail. So, A is incorrect.Option B: Fixed cost is the cost of establishment of plant and machinery. Average fixed cost is equal to total fixed cost divided by total output. As output increases, AFC decreases. So, B is incorrect.Option C: As firms produce, the productivity of capital goods (machinery) keeps declining as more and more workers are added to the same fixed set of machineries. Since machineries can’t be added overnight, the productivity of capital falls with increasing production, unless new machinery is installed in the long-run.Q Source: Improvisation: UPSC CDS 201342. Consider the following statements with reference todevelopments in Medieval India. 1. A personal astronomical observatory was built by Humayun on the banks of Yamuna River. 2. Metal cylinder rockets were often used in wars waged by some of the Mughal rulers. 3. Shampoo bath techniques and related chemicals wereintroduced in Mughal India from Europe.Select the correct answer using the codes below.a) 1 and 2 only b) 2 and 3 only c) 1 only d) 1, 2 and 3Solution: a)Justification: Statement 1: Humayun was absorbed in astrology and astronomy.Gyarah Sidi are the remains of the astrological observatory of the Mughal Emperor Humayun.The ruins are situated at a stone’s throw from Babur’s Mehtab Bagh, in a field on the banks of the Yamuna River in Agra.Statement 2: Other Technological developments too occurred during Mughal Empire.• Fathullah Shirazi (1582), a Persian polymath and mechanical engineer who worked for Akbar, developed a volley gun.• Akbar was the first to initiate and use metal cylinder rockets known as bans particularly against War elephants, during the Battle of Sanbal.• Later, the Mysorean rockets were upgraded versions of Mughal rockets used during the Siege of Jinji by the progeny of the Nawab of Arcot.Statement 3: Sake Dean Mahomed had learned much of Mughal alchemy and understood the techniques used to produce various alkali and soaps to produce shampoo when he was in India. He introduced South Asian cuisine and shampoo baths to Europe, where he offered therapeutic massage.

Page 10: stru file · Web view1. Consider the following about Ibn Battuta’s account of India. 1. The highways were safe from robbery and other crimes. 2. Coconut fibre found in India was

Sake Dean Mahomed was appointed as shampooing surgeon to both Kings George IV and William IV !Q Source: Improvisation: Chapter 8: Themes in Indian History – II: 12th NCERT43. Heritage Heroes Award is given by which of the followingorganizations? a) International Union for Conservation of Nature (IUCN) b) Greenpeace c) United Nations Educational, Scientific and CulturalOrganization (UNESCO) d) World Wildlife Fund (WWF)Solution: a)Learning: Assam-based ecologist and conservation activist Bibhuti Lahkar has become the first Asian to be awarded the prestigious Heritage Heroes Award by the International Union for Conservation of Nature (IUCN).He received the award at the IUCN’s World Conservation Congress.He has been working to save the grasslands, flora and fauna of Manas National Park area for the past two decades.He was instrumental in connecting Manas Wildlife Sanctuary with the Royal Manas National Park in Bhutan.He had also conducted a GIS survey of the Manas area and his research findings were highly beneficial in the Manas Tiger Conservation.Q Source: http://www.business-standard.com/article/pti- stories/ecologist-bibhuti-lahkar-bags-heritage-heroes-award-of-iucn- 116090400498_1.html44. Consider the following about rural institutions in MedievalIndia. 1. The village panchayats represented only the most dominant caste in the village. 2. The village headman was to be compulsorily nominated by the Subedar who headed the Parganas. 3. The panchayat could derive its funds from contributions madeby community residents to a common financial pool.Select the correct answer using the codes below.a) 1 and 2 only b) 3 only c) 1 and 3 only d) 2 and 3 onlySolution: b)Justification: Statement 1: The village panchayat was an assembly of elders. In mixed-caste villages, the panchayat was usually a heterogeneous body. The panchayat represented various castes and communities in the village so it is called an oligarchy.• In addition to the village panchayat each sub-caste or jati in the village had its own jati panchayat.• These panchayats wielded considerable power in rural society.• In Rajasthan jati panchayats arbitrated civil disputes between members of different castes.Statement 2: The panchayat was headed by a headman known as muqaddam or mandal. Some sources suggest that the headman was chosen through the consensus of the village elders, and that this choice had to be ratified by the zamindar. Headmen held office as long as they enjoyed the confidence of the village elders.Statement 3: The chief function of the headman was to supervise the preparation of village accounts, assisted by the accountant or patwari of the panchayat.The panchayat derived its funds from contributions made by individuals to a common financial pool.Q Source: Page 201: Themes in Indian History – II: 12th NCERT45. INS Chennai is a/ana) Stealth Guided missile destroyer ship b) Submarine torpedo launcher c) Naval Aircraft carrier d) Surface dock travellerSolution: a)Learning: INS Chennai, a Kolkata-class stealth Guided missile destroyer ship was commissioned into Indian Navy’s combat fleet.It is the largest-ever warship to be built in India. It is third and last Kolkata-class guided missile destroyers built under Project 15A. Thus, its induction also marks the end of the Project 15A.The ship has enhanced stealth features resulting in a reduced Radar Cross Section (RCS) and uses radar transparent materials on exposed decks.It has undersea warfare capabilities.Q Source: http://www.business-standard.com/article/government- press-release/guided-missile-destroyer-ins-chennai-joins-the-indian- navy-116112100895_1.html46. Consider the following International Film Festival of India(IFFI). 1. It is held annually in Goa. 2. It is organized by the Film and Television Institute of India (FTII). 3. South Korea is the focus country of IFFI 2016.Select the correct answer using the codes below.a) 1 only b) 1 and 2 only c) 1 and 3 only d) 2 and 3 onlySolution: c)Justification: Statement 1: The 47th edition of the International Film Festival of India (IFFI) began in Goa. It is one of Asia’s oldest and India’s biggest film festival (since 1952).Statement 2: It is the biggest state-backed film festival and conducted jointly by Union Ministry of Information and Broadcasting, Directorate of Film Festivals and Government of Goa.It aims at providing a common platform for the cinemas of the world to project the excellence of the film art.It also seeks to contribute to create awareness for understanding and appreciation of film cultures of different nations in the context of their social and cultural ethos and promote friendship and cooperation among people of the world.Statement 3: South Korea is the focus country of IFFI 2016. Polish film “After-Image” of filmmaker Andrzej Wajda’s is the opening film of the festival. The South Korean film “Age of Shadows” of Kim Jee Woon will be the festival’s closing film. This film also South Korea’s official entry for the Oscar awards.Q Source: IFFI 201647. Consider the following about the Khilafat Movement.1. It was initiated by Maulana Abdul Kalam Azad. 2. It demanded that Khalifa must retain control over the Muslimsacred places in the erstwhile Ottoman Empire. 3. The Congress boycotted the movement due to its violentmoorings.Select the correct answer using the codes below.a) 1 and 2 only b) 2 and 3 only c) 2 only d) 1 and 3 onlySolution: c)Justification: Statement 1: The Khilafat Movement, (1919-1920) was a movement of Indian Muslims, led by Muhammad Ali and Shaukat Ali.Statement 2: It demanded the following: The Turkish Sultan or Khalifa must retain control over the Muslim sacred places in the erstwhile Ottoman empire; the jazirat-ul-Arab (Arabia, Syria, Iraq, Palestine) must

Page 11: stru file · Web view1. Consider the following about Ibn Battuta’s account of India. 1. The highways were safe from robbery and other crimes. 2. Coconut fibre found in India was

remain under Muslim sovereignty; and the Khalifa must be left with sufficient territory to enable him to defend the Islamic faith.Statement 3: The Congress supported the movement and Mahatma Gandhi sought to conjoin it to the Non-cooperation Movement.Q Source: Page 350: Themes in Indian History – III: 12th NCERT48. Consider the following statements.1. Gandhiji spent a year travelling around British India on Gokhale’s advice. 2. Gandhiji first forged the techniques of non-violent satyagraha inSouth Africa.Which of the above is/are correct?a) 1 only b) 2 only c) Both 1 and 2 d) NoneSolution: c)Justification: Statement 1: He wanted to know the land of India and its peoples, and hence decided to travel across India on the advice of Gokhale.Such statements are often asked by UPSC (e.g. see Q on Home Rule Movement CSP 2015 or Q on Champaran Satyagraha in CAPF 2016) despite their inherent subjectivity.His first major public appearance was at the opening of the Banaras Hindu University (BHU) in February 1916. We have covered the speech in the previous test.Statement 2: In 1915, Gandhi returned to his homeland after two decades of residence abroad. These years had been spent for the most part in South Africa, where he went as a lawyer, and in time became a leader of the Indian community in that territory.As the historian Chandran Devanesan has remarked, South Africa was “the making of the Mahatma”. It was in South Africa that Mahatma Gandhi first forged the distinctive techniques of non-violent protest known as satyagraha, first promoted harmony between religions, and first alerted upper-caste Indians to their discriminatory treatment of low castes and women.Q Source: Page 348: Themes in Indian History – III: 12th NCERT49. You would recommend which of these measures during aneconomic recession? 1. Giving a fiscal stimulus 2. Increase tax rates 3. Lower interest ratesSelect the correct answer using the codes below.a) 1 only b) 2 only c) 1 and 3 only d) 1, 2 and 3Solution: c)Justification: Statement 1: Fiscal stimulus means a package of large government spending. It boosts aggregate demand, revives consumer spending and increases investment via creation of infrastructure.Statement 2: High tax rates would reduce disposable income and further reduce demand for consumer goods leading to an even pronounced slowdown. So, 2 is incorrect.Statement 3: Lower interest rates will encourage investors to borrow and invest in new projects thereby enhancing aggregate demand and employment uplifting the economy.Q Source: Improvisation: UPSC CDS 201350. The following are some of the famous books written bypopular travellers to India. 1. Rihla: Francois Bernier 2. Kitab-ul-Hind: Al-Biruni 3. Ain-i-Akbari: Abdur Razzaq 4. Matla-us-Sadain: Seydi Ali ReisSelect the correct matches using the codes below.a) 1 and 2 only b) 2 and 4 only c) 1, 3 and 4 only d) 2 onlySolution: d)Justification: Statement 1: Ibn Battuta’s book of travels, called Rihla, written in Arabic. His account is often compared with that of Marco Polo, who visited China (and also India) from his home base in Venice in the late thirteenth century.Statement 2: Al-Biruni travelled widely in the Punjab and parts of northern India. He collected various information and wrote a book called Kitab-ul-Hind.It deals with religion and philosophy, festivals, astronomy, alchemy, manners and customs, social life, weights and measures, iconography, laws and metrology.Statement 3: It was written by Abul Faizl. It was an administrative account of the state of Akbar.Statement 4: Abd-ur-Razzaq's narrative of his visit to India includes describing the life and events in Calicut under the Zamorin and also of the Ancient City of Vijayanagara at Hampi, describing their wealth and immense grandeur.Abd-ur-Razzaq's Matla-us-Sadain wa Majma-ul-Bahrain also included a detailed account of diplomatic relations between Shah Rukh's state and the Ming China.Q Source: Pages - Multiple: Chapter 7 and 8: Themes in Indian History – II: 12th NCERT51. The largest consumer of ground water in India isa) Irrigation sector b) Textiles sector c) Household sector d) Commercial Reverse Osmosis (RO) plantsSolution: a)Learning: In general, the irrigation sector remains the main consumer of ground water (nearly 90% of total annual ground water draft for all uses).India is the largest user of groundwater in the world. It uses an estimated 230 cubic kilometers of groundwater per year - over a quarter of the global total.More than 60% of irrigated agriculture and 85% of drinking water supplies are dependent on groundwater.As per the World Bank, if current trends continue, in 20 years about 60% of all India’s aquifers will be in a critical condition.Q Source: Chapter 6: 12th NCERT: India, People and Economy52. Agricultural lands are generally left as fallow lands. This isdone because a) Soil gets time to recoup its natural fertility. b) Organic matter in soil exceeds the desired levels. c) Fallow lands are converted to wastelands to construct embankments. d) These lands are left to be used for social forestry.Solution: a)Learning: Fallow is the stage of crop rotation in which the land is deliberately not used to raise a crop.Ground may be fallowed as part of a larger crop rotation plan or as a method to conserve moisture as in the summer fallow technique used in dryland farming.The practice of leaving fields fallow dates back to ancient times when farmers realized that using soil over and over again depleted its nutrients.If the duration of fallow is substantially reduced owing to multiple cropping, the soil does not get enough time to recuperate and allow natural fertilization like nitrogen fixation. If fallow is increased, it will help in nutrient replenishment.

Page 12: stru file · Web view1. Consider the following about Ibn Battuta’s account of India. 1. The highways were safe from robbery and other crimes. 2. Coconut fibre found in India was

Q Source: Improvisation: Page 58: 12th NCERT: India, People and Economy53. Consider the following about the Vijayanagara Empire. 1. Chinese porcelains were banned within the Empire by therulers.2. Arrangements were made to store rainwater and conduct it to the city. 3. Fortifications around the Empire enclosed agricultural tracts.Select the correct answer using the codes below.a) 1 and 2 only b) 2 and 3 only c) 1 only d) 1, 2 and 3Solution: b)Justification: Statement 1: Archaeologists have found fine Chinese porcelain in some areas of the Empire, which suggest that these areas may have been occupied by rich traders. Statement 1 is wrong.Statement 2: The most striking feature about the location of Vijayanagara is the natural basin formed by the river Tungabhadra which flows in a north-easterly direction.Water from this tank not only irrigated fields nearby but was also conducted through a channel to the “royal centre”. One of the most prominent waterworks to be seen among the ruins is the Hiriya canal.Statement 3: Abdur Razzaq, an ambassador sent by the ruler of Persia was greatly impressed by the fortifications, and mentioned seven lines of forts. These encircled not only the city but also its agricultural hinterland and forests.• Abdur Razzaq quoted that between the first, second and the third walls there are cultivated fields, gardens and houses.• The objective of medieval sieges was to starve the defenders into submission. These sieges could last for several months and sometimes even years.• Normally rulers tried to be prepared for such situations by building large granaries within fortified areas.Q Source: Page 177: Themes in Indian History – II: 12th NCERT54. Consider the following statements.1. Coastal lagoons generally form near flat or gently sloping landscapes. 2. The size and depth of coastal lagoons often depend on sea level.Which of the above is/are correct?a) 1 only b) 2 only c) Both 1 and 2 d) NoneSolution: c)Justification: A lagoon is a shallow body of water protected from a larger body of water (usually the ocean) by sandbars, barrier islands, or coral reefs. Lagoons are often called estuaries, sounds, bays, or even lakes.Statement 1: Lagoons sheltered by sandbars or barrier islands are called coastal lagoons. Coastal lagoons form along coastal plains—flat or gently sloping landscapes. They form in areas with small tidal ranges. Coastal lagoons are created as a shallow basin near the shore gradually erodes, and the ocean seeps in between the sandbars or barrier islands.Statement 2: The size and depth of coastal lagoons often depend on sea level. When the sea level is low, coastal lagoons are swampy wetlands. When the sea level is high, they can look like coastal lakes or bays.Q Source: Improvisation: Page 63: 12th NCERT: India, People and Economy55. Consider the following about Alvars and Nayanars.1. They found the caste system valid and based on practical lines. 2. Women devotees were not allowed within the Sangha.Select the correct answer using the codes below.a) 1 only b) 2 only c) Both 1 and 2 d) NoneSolution: d)Justification: Statement 1: Alvars and Nayanars seem to have initiated a movement of protest against the caste system and the dominance of Brahmanas. Other historians point that they at least attempted to reform the system.To some extent this is corroborated by the fact that bhaktas hailed from diverse social backgrounds ranging from Brahmanas to artisans and cultivators and even from castes considered “untouchable”.Statement 2 & 3: The compositions of Andal, a woman Alvar, were widely sung. Another woman, Karaikkal Ammaiyar, a devotee of Shiva, adopted the path of extreme asceticism in order to attain her goal. So, 2 is wrong.Q Source: Page 144: Themes in Indian History – II: 12th NCERT56. The Deccan Riots Commission was appointed toa) Frame charges against those complicit in the 1857 mutiny b) Revamp the police machinery to tackle increasing riots in the Deccan region c) Initiate proceedings against guilty Zamindars for transacting benami properties d) Investigate into the causes of the Deccan uprisingSolution: d)Learning: The Deccan peasants uprising 1875 was directed mainly against the excesses of the Marwari and Gujarati money lenders.• Social boycott of moneylenders by the peasants was later transformed into armed peasant revolt in the Poona and Alimadnagar districts of Maharashtra.• By June 1875 nearly a thousand peasants were arrested and the uprising completely suppressed.• The Government appointed the Deccan Riots Commission to investigate into the causes of the uprising.• The ameliorative measure passed was the Agriculturists Relief Act of 1879 which put restrictions on the operations of the peasants land and prohibited imprisonment of the peasants of the Deccan for failure to repay debts to the moneylenders.Q Source: Page 285: Themes in Indian History – III: 12th NCERT57. Consider the following about the Dharwar System of rocks.1. It is the oldest rock system of India. 2. It is contains metamorphic sedimentary rocks. 3. It hosts gold mines.Select the correct answer using the codes below.a) 1 and 2 only b) 2 only c) 1 and 3 only d) 2 and 3 onlySolution: d)Justification: Statement 1: Dharwar system is later than the Archean system but older than the other systems. The Dharwar period of rockformation has been largely fixed from 2500 million years ago to 1800 million years ago.Statement 2: Dharwar Rock System is special because it is the first metamorphic sedimentary rocks in India. They are named Dharwar system because they were first studied in Dharwar region of Karnataka. But they are also found in Aravallis, Tamil Nadu, Chotanagpur plateau, Meghalaya, Delhi, and the Himalayas region.

Page 13: stru file · Web view1. Consider the following about Ibn Battuta’s account of India. 1. The highways were safe from robbery and other crimes. 2. Coconut fibre found in India was

Statement 3: The Dharwar rocks are rich in iron ore, manganese, lead, zinc, gold, silver etc. The Champions series containing gold mines lie within this system. This Champion system is named after the Champion reef in the Kolar Gold Fields. The Kolar Gold Fields contain one of the deepest gold mines of world.Q Source: Improvisation: Chapter 7: 12th NCERT: India, People and Economy58. The Ahom kings followed a peculiar practice where people were obliged to render military service in exchange for land. The practice was popular in the present day state of a) Meghalaya b) Nagaland c) Sikkim d) AssamSolution: d)Learning: Tribes in the North-east had their chieftains. Many tribal chiefs had become zamindars, some even became kings. For this they required to build up an army.They recruited people from their lineage groups or demanded that their fraternity provide military service. Tribes in the Sind region had armies comprising 6,000 cavalry and 7,000 infantry.These people who exchanged military service for land were known as paiks in the Ahom dynasty. The capture of wild elephants was declared aroyal monopoly by the Ahom kings. This was because war was a common occurrence between tribal kingdoms in the north-east.Q Source: Page 210: Themes in Indian History – II: 12th NCERT59. Consider the following statements.Assertion (A): In some parts of Southern India, Kharif crops can be grown during any period in the year.Reason (R): Southern India receives continuous but light rainfall from western disturbances throughout the year.In the context of the above, which of these is correct?a) A is correct, and R is an appropriate explanation of A. b) A is correct, but R is not an appropriate explanation of A. c) A is correct, but R is incorrect. d) Both A and R are incorrect.Solution: a)Justification: The distinction between Kharif, Rabi and Zaid seasons is not as marked in South India as it in Northern India, and tropical crops can be grown during any period in the year in Southern India.This is because the temperature in South India is generally high enough to grow these crops during any part of the year.So, same crops can be grown thrice in an agricultural year provided there is sufficient soil moisture. So, A is correct.Western disturbances appear in the winter season in India and withdraw when Monsoon comes. They are not responsible for majority of rainfall in South India. They become more important in the Northern regions. So, R is incorrect.Q Source: Page 47: 12th NCERT: India, People and Economy60. Consider the following statements.1. Expansion of foreign trade in the Mughal Empire often involved the use of silver bullions as payment. 2. Silver coins were discontinued in British India in view ofgrowing deficit of the precious metal.Which of the above is/are correct?a) 1 only b) 2 only c) Both 1 and 2 d) NoneSolution: a)Justification: Statement 1: Voyages of discovery and the opening up of the New World (America) resulted in a massive expansion of India’s trade with Europe.• An expanding trade brought in huge amounts of silver bullion into India to pay for goods procured from India. This was good for India because it did not have natural resources of silver.• As a result, the period between the 16th and 17th centuries was also marked by a remarkable stability in the availability of metal currency, particularly the silver rupya in India. This facilitated an unprecedented expansion of minting and circulating of silver coins.Statement 2: Copper, Gold and Silver coinages were issued during the British rule.There are many rare coins of this period which interests the coin collectors. The 1939 Rupee is the most expensive rupee, as after 1939 all silver coins effectively became less pure, due to the shortage of silver during the world war.Q Source: Improvisation: Chapter 8: Themes in Indian History – II: 12th NCERT61. The Bardoli Satyagraha of 1928 was a/ana) A no-tax movement of peasants b) Anti land leasing movement led by Zamindars c) Reaction against Commercialization of agriculture d) Frenzied outburst of tribals against land encroachment by theBritishSolution: a)Learning: In 1925, the taluka of Bardoli in Gujarat suffered from floods and famine, causing crop production to suffer and leaving farmers facing great financial troubles.• However, the government of the Bombay Presidency had raised the tax rate by 30% that year, and despite petitions from civic groups, refused to cancel the rise in the face of the calamities.• Patel first wrote to the Governor of Bombay, asking him to reduce the taxes for the year in face of the calamities. But the Governor ignored the letter, and reciprocated by announcing the date of collection.• This provocation by the government led to a major episode of civil disobedience and revolt in the Indian Independence Movement.• The movement was eventually led by Vallabhbhai Patel, and its success gave rise to Patel becoming one of the main leaders of the independence movement.Q Source: Improvisation: Page 355: Themes in Indian History – II: 12th NCERT62. Consider the following about sulh-i-kul introduced duringAkbar’s regime. 1. The ideal of sulh-i kul was implemented through state policies. 2. Sulh-i-kul allowed all religions and schools of thought toflourish provided they don’t impinge on state authority.Which of the above is/are correct?a) 1 only b) 2 only c) Both 1 and 2 d) NoneSolution: c)Justification: Statement 1: Akbar abolished the tax on pilgrimage in 1563 and jizya in 1564 as the two were based on religious discrimination.All Mughal emperors gave grants to support the building and maintenance of places of worship. Even when temples were destroyed during war, grants were later issued for their repair.

Page 14: stru file · Web view1. Consider the following about Ibn Battuta’s account of India. 1. The highways were safe from robbery and other crimes. 2. Coconut fibre found in India was

Statement 2: In sulh-i kul all religions and schools of thought and freedom of expression but on condition that they did not undermine the authority of the state or fight among themselves. This was in line with Akbar’s view that all religions point towards the same thing – the essential unity among human beings.Q Source: Page 234: Themes in Indian History – II: 12th NCERT63. Open burning of coal can expose you to1. Beryllium 2. Mercury 3. Sulfur dioxide 4. Nitrogen oxidesSelect the correct answer using the codes below.a) 1 and 3 only b) 2, 3 and 4 only c) 1 and 4 only d) 1, 2, 3 and 4Solution: d)Justification: Burning coal is also a leading cause of smog, acid rain (SOx and NOx), and toxic air pollution.Coal plants are responsible for more than half of the U.S. human-caused emissions of mercury.Other harmful pollutants emitted annually from an uncontrolled coal plant include lead, cadmium, other toxic heavy metals, carbon monoxide, arsenic, volatile organic compounds (VOC) etc.Inhalation exposure to beryllium primarily occurs in the workplaces where it is mined, processed, or converted into alloys and chemicals, or from the burning of coal or fuel oil and in tobacco smoke.Q Source: Improvisation: Page 136: 12th NCERT: India, People and Economy64. Consider the following with reference to the MultilateralAutomatic Exchange of Financial Account Information (MCAA). 1. It is a multilateral convention on administrative assistance in taxation matters. 2. It was developed by the Organisation for Economic Co- operation and Development (OECD). 3. India has not joined the convention.Select the correct answer using the codes below.a) 1 and 2 only b) 2 and 3 only c) 1 onlyd) 1, 2 and 3Solution: a)Justification: It sets up a system wherein bulk taxpayer information will be periodically be sent by source country of income to the country of residence of the taxpayer. So, 1 is correct.• This agreement obliges signatories to exchange a wide range of financial information among themselves periodically and automatically.• Its main aim is to prevent international tax evasion and avoidance and help member countries to curb tax evasion and deal with the problem of black money.• It was signed by 53 jurisdictions in 2015 based on Article 6 of the Convention on Mutual Administrative Assistance in Tax Matters. India too had joined it in 2015. So, 3 is wrong.Q Source: http://pib.nic.in/newsite/PrintRelease.aspx?relid=122256http://www.business-standard.com/article/government-press- release/india-and-switzerland-sign-joint-declaration-for-the- implementation-of-automatic-116112201068_1.html65. In the context of Modern History of India, the AzamgarhProclamation is generally associated with a) Sepoy Mutiny 1857 b) Swadeshi Movement c) Rowlatt Satyagraha d) Champaran MovementSolution: a)Learning: This proclamation was published in the Delhi Gazette in the midst of the “Great Mutiny” of 1857.• The author was most probably Firoz Shah, a grandson of the Mughal emperor Bahadur Shah Zafar, whose restoration to full power was a main aim of the rebels.• It expressed complete disillusionment with the British Rule and express the fear that British missionaries were, with government connivance, attempting to Christianize India came to a head among the British East India Company’s sepoy troops.• It is one of the most significant sources of information about the objective of the rebels.Q Source: Page 301: Themes in Indian History – III: 12th NCERT66. The Karachi Session of the Indian National Congress isfamously known for 1. Adopting the Nehru report 2. A concrete Fundamental Rights and Economic Programme 3. Endorsing the Gandhi Irwin PactSelect the correct answer using the codes below.a) 1 and 2 only b) 2 and 3 only c) 3 only d) 2 onlySolution: b)Justification: Statement 1: The draft constitution of India was called “Nehru Committee Report“. This report was submitted in 1928 at the Lucknow conference of all the parties, where Jinnah opposed it.In the Calcutta Session 1929 of INC, the Nehru Report was accepted by a majority vote. The congress gave an ultimatum to the British Government to accept the recommendation of the report by the end of 1929, and also threatened for another mass movement in case the report is not accepted. The report was not accepted by the Government.Statement 2: Some important aspects of these resolutions were:• Basic civil rights of freedom of speech, Freedom of Press, Freedom of assembly, Freedom of association, Equality before law• Elections on the basis of Universal Adult Franchise• Free and compulsory primary education• Substantial reduction in rent and taxes• Better conditions for workers including a living wage, limited hours of work• Protection of women and peasants Government ownership or control of key industries, mines, and transport• Protection of Minorities.Statement 3: Bhagat Singh, Sukhdev and Rajguru were executed a week before the Pact was endorsed. So, there was anger in the public why Gandhi signed the pact.The details of the pact have been mentioned in another question in this test.Q Source: Improvisation: Page 360: Themes in Indian History – III: 12th NCERT67. Which of these social schemes/programmes has a bearing ontribal welfare? 1. Mahatma Gandhi National Rural Employment Guarantee Act (MGNREGA) 2. Deen Dayal Antyodaya Yojana 3. Pradhan Mantri Gram Sadak Yojana (PMGSY) 4. Stand Up India SchemeSelect the correct answer using the codes below.

Page 15: stru file · Web view1. Consider the following about Ibn Battuta’s account of India. 1. The highways were safe from robbery and other crimes. 2. Coconut fibre found in India was

a) 1, 2 and 3 onlyb) 2 and 4 only c) 1 and 4 only d) 1, 2, 3 and 4Solution: d)Justification: Statement 1: MGNREGA provides the right to work to every adult living in rural areas. This applies to tribals as well; hence the bearing on tribal welfare.Statement 2: It is the revised version of NRLM (Aajevika) yojana that aims to reduce rural poverty by empowering Self-Help groups (SHG).Statement 3: PMGSY has connected many tribal hamlets, previously unconnected by an all-weather road.Statement 4: Stand up India schemes specially targets SC/ST and women entrepreneurs.Q Source: Improvisation: Page 106: 12th NCERT: India, People and Economy68. Consider the following statements.1. Portuguese travellers Domingo Paes and Fernao Nuniz visited his Empire. 2. The south Indian mathematician Nilakantha Somayaji lived inhis Empire.The above refers to?a) Pratipala b) Navdevya Mahismati c) Rajaraja I d) KrishnadevarayaSolution: d)Learning: Krishnadeva Raya’s rule was characterised by expansion and consolidation.• This was the time when the land between the Tungabhadra and Krishna rivers (the Raichur doab) was acquired (1512), the rulers of Orissa were subdued (1514) and severe defeats were inflicted on the Sultan of Bijapur (1520).• Although the kingdom remained in a constant state of military preparedness, it flourished under conditions of unparalleled peace and prosperity.• Krishnadeva Raya is credited with building some fine temples and adding impressive gopurams to many important south Indian temples.• He also founded a suburban township near Vijayanagara called Nagalapuram after his mother. Some of the most detailed descriptions of Vijayanagara come from his time or just after.Q Source: Page 173: Chapter 7: Themes in Indian History – II: 12th NCERT69. Which of the following will be the main agency handling the establishment of National Cyber Coordination Centre (NCCC)? a) The Cyber Society of India (CySI) b) Ministry of Home Affairs (MoHA) c) Indian Computer Emergency Response Team (CERT-In) d) National Informatics Centre (NIC)Solution: c)Learning: It is a proposed cyber security and e-surveillance agency in India.• Some of the components of NCCC include a cyber crime prevention strategy, cyber crime investigation training, review of outdated laws, etc.• Indian and U.S. intelligence agencies are also working together to curb misuse of social media platforms in the virtual world by terror groups.• Some have expressed concern that the body could encroach on Indian citizens' privacy and civil-liberties, given the lack of explicit privacy laws in the country.• This Centre will have top experts from the field and it will be run like similar organisation in other countries such as the US, the UK, France, Germany, etc.Q Source: http://pib.nic.in/newsite/PrintRelease.aspx?relid=13389570. Gandhi opposed separate electorates because he believed1. It would mean perpetual bondage of the harijans. 2. It would lead to social disharmony and lack of unity.Which of the above is/are correct?a) 1 only b) 2 only c) Both 1 and 2 d) NoneSolution: c)Justification: Statement 1: Separate Electorates mean the community to which the electorate belongs would choose their own leaders via an election in which the candidates of only their community would be allowed to contest and only their community members would vote.This would mean that the elections for choosing the leaders of that particular community would be held separately and would not come under general elections.He believed this system will institutionally cement the untouchable status of harijans forever.Statement 2: Gandhi, who was in jail in Poona, declared, “I am certain that the question of separate electorates for the Untouchables is the modern manufacture of satanic government. I will resist it with my life.” He was opposed to separate electorates and saw three Indias: Hindus, Muslims, and Untouchables.Q Source: Page 360: Themes in Indian History – II: 12th NCERT71. The amara-nayakas in the Vijayanagara Empire werea) Royal spies who delivered information from one part of empire to another b) Hindu Diwans who were given the title of state landlords c) Scholars tasked with interpreting ancient religious texts d) Military commanders who were handed over territories togovernSolution: d)Learning: The amara-nayaka system was a major political innovation of the Vijayanagara Empire. It is likely that many features of this system were derived from the iqta system of the Delhi SultanateAmara-nayakas collected taxes and other dues from peasants, craftspersons and traders in the area.The amara-nayakas sent tribute to the king annually and personally appeared in the royal court with gifts to express their loyalty.They retained part of the revenue for personal use and for maintaining a stipulated contingent of horses and elephants.These contingents provided the Vijayanagara kings with an effective fighting force with which they brought the entire southern peninsula under their control.Q Source: Page 175: Themes in Indian History – II: 12th NCERT72. She is the “First woman in the World” to receiveInternational Maritime Organisation (IMO) award for Exceptional Bravery at Sea a) Lakshmi Sehgal b) Radhika Menon c) Sita Sahu d) Aruna Asaf AliSolution: b)Learning: Recently, Captain Radhika Menon was awarded with the International Maritime Organisation (IMO) award for Exceptional Bravery at Sea.With this, she became the first woman in the world to receive the Award.She was presented with this award for saving lives of seven fishermen from sinking fishing boat in Bay of Bengal.In 2011, Radhika Menon had made history by becaming the first woman to become captain of Indian Merchant Navy.

Page 16: stru file · Web view1. Consider the following about Ibn Battuta’s account of India. 1. The highways were safe from robbery and other crimes. 2. Coconut fibre found in India was

International Maritime Organisation (IMO): It UN’s specialised agency responsible for the safety and security of shipping and the prevention of marine pollution by ships.Q Source: http://www.thehindubusinessline.com/economy/logistics/bravery- award-for-indias-first-female-merchant-navy- captain/article9374744.ece73. The South-Western Plateau Region is known for the depositsof which of the following minerals?1. Ferrous metals 2. Bauxite 3. Coal 4. KyaniteSelect the correct answer using the codes below.a) 3 and 4 only b) 1 and 2 only c) 1, 3 and 4 only d) 1, 2, 3 and 4Solution: d)Learning: This belt extends over Karnataka, Goa and contiguous Tamil Nadu uplands and Kerala. This belt is rich in ferrous metals and bauxite.India is the largest producer of kyanite in the world. The USA, the UK and Japan depend heavily on imports from India. Jharkhand, Maharashtra and Karnataka produce practically the whole of kyanite of India.• Kyanite in the form of mullite is widely used in the manufacture of glass, burner tips, spark plugs, heating elements and high voltage electrical insulations and in the ceramic industry.The belt also contains high grade iron ore, manganese and limestone. This belt packs in coal deposits except Neyveli lignite.This belt does not have as diversified mineral deposits as the north- eastern belt.Kerala has deposits of monazite and thorium, bauxite clay. Goa has iron ore deposits.Q Source: Page 73: 12th NCERT: India, People and Economy74. In Medieval India, Muqaddams werea) Foreign travellers who were given imperial responsibilities b) Quazis who issued decree on behalf of the Emperor c) Caretakers of the luxury estates of the royal family in different parts of the Empire d) Headmen of villageSolution: d)Learning: As per the Persian documents of medieval India, a muqaddam was the headman of a village.• He was, by profession, a peasant of the village which he headed. He could sell and buy land for the village and settle the common treasury.• His position was hereditary; however, it could also be bought and sold. He was never a government servant, but he could be dispossessed of his status by the revenue official• Some sources suggest that the headman was chosen through the consensus of the village elders, and that this choice had to be ratified by the zamindar.• Headmen held office as long as they enjoyed the confidence of the village elders, failing which they could be dismissed by them.• The chief function of the headman was to supervise the preparation of village accounts, assisted by the accountant or patwari of the panchayat.Q Source: Page 202: Chapter 8: Themes in Indian History – II: 12th NCERT75. Consider the following about the Indian Council for CulturalRelations (ICCR). 1. It was founded before India’s independence. 2. It aims to promote cultural exchange with other countries and peoples. 3. It funds research in premier cultural institutions of India.Select the correct answer using the codes below.a) 2 only b) 1 and 3 only c) 1 and 2 only d) None of the aboveSolution: a)Justification: Statement 1: It was founded in 1950 by Maulana Abul Kalam Azad, independent India’s first Education Minister.Statement 2: Its objectives are to actively participate in the formulation and implementation of policies and programmes pertaining to India’s external cultural relations; to foster and strengthen cultural relations and mutual understanding between India and other countries; to promote cultural exchanges with other countries and people; and to develop relations with nations.Q Source: http://pib.nic.in/newsite/PrintRelease.aspx?relid=136740http://www.mea.gov.in/indian-council-for-cultural-relations.htm76. The British government convened a series of “Round TableConferences” in London to a) Chart out a long-term economic plan for India b) Address the grievances of the capitalist class of both India and Britain c) Discuss constitutional reforms in India d) Review major trade and investment policies of the British inIndiaSolution: c)Learning: Three Roundtable Conferences were held between 1930 and 1932 by the British Government to discuss constitutional reforms in India, with the ever growing demands of self-rule.• The first meeting was held in November 1930, but without the pre- eminent political leaders of India.• A second Round Table Conference was held in London in the latter part of 1931. Congress representative attended the conference. However, Congress’ claims that the party represented all of India came under challenge from three parties: from the Muslim League, which claimed to stand for the interests of the Muslim minority; from the Princes, who claimed that the Congress had no stake in their territories; and from B.R. Ambedkar, who argued that Congress did not really represent the lowest castes.• The Conference in London was inconclusive, so Gandhiji returned to India and resumed civil disobedience.Q Source: Page 360: Themes in Indian History – II: 12th NCERT77. The Reserve Bank of India (RBI) has proposed the opening ofIslamic window in conventional banks to give a fillip to a) Self-Help Group (SHG) Bank Linkage Programme b) Minority specific accounts c) Micro-finance Institutions d) Interest-free bankingSolution: d)Learning: Islamic or Sharia banking is a finance system based on the principles of not charging interest.The charging of interest is prohibited under Islam.• Recently both the Union Government and the RBI have been exploring the possibility of introduction of Islamic banking for a while to ensure financial inclusion of those sections of society that have remain excluded due to religious reasons.• Initially, RBI is planning to introduce a few simple products similar to conventional banking products through the Islamic window. But, it will be necessary after the notification is issued by the Union Government.• In later stage, full-fledged Islamic banking with profit-loss sharing complex products may be considered on the basis of experience gained.

Page 17: stru file · Web view1. Consider the following about Ibn Battuta’s account of India. 1. The highways were safe from robbery and other crimes. 2. Coconut fibre found in India was

Q Source: http://indianexpress.com/article/business/banking-and- finance/rbi-proposes-islamic-window-in-banks-for-gradual- introduction-of-sharia-banking-4385584/78. India trades the most in terms of market value with which ofthese groups or organizations? a) European Union b) Gulf Cooperation Council c) ASEAN d) SAARCSolution: b)Learning: As a single trading partner, GCC is our largest partner, with almost $160 billion trade. If we take into account our trade with two other countries in the Gulf region, Iraq ($21.5 billion) and Iran ($15 billion) the trade with the region becomes $195 billion. Additionally we get $30 billion annually as remittances from the GCC countries.Add all this up and our economic relationship with the region becomes $225 billion. This is more than the combined trade with ASEAN and China; or entire EU and US.The European Union is India's second largest trading bloc, accounting for around 20% of Indian trade.Some of India's largest trading partner nations are the United Arab Emirates, China, the United States, Saudi Arabia, Switzerland, Singapore, Germany, Hong Kong, Indonesia, Iraq and Japan.Q Source: Improvisation: Page 129: 12th NCERT: India, People and Economy79. Consider the following statements about the recentdevelopments with reference to the International Criminal Court (ICC). 1. India and China have applied for the membership of the Rome statue which governs the ICC. 2. Recently Russia has officially declared its withdrawal from theRome Statue and ICC.Which of the above is/are correct?a) 1 only b) 2 only c) Both 1 and 2 d) NoneSolution: b)Justification: Statement 1: Currently, there are 124 states which are party to Rome Statute and therefore members of the ICC (India and China are not its members). They haven’t applied for the membership too.The government of India has consistently opposed the Court. It abstained in the vote adopting of the statute in 1998, saying it objected to the broad definition adopted of crimes against humanity; the rights given to the UN Security Council to refer and delay investigations and bind non-states parties; and the use of nuclear weapons and other weapons of mass destruction not being explicitly criminalized.Statement 2: The executive order mentioned that Russia is pulling out of the 2002 Rome Statute, which establishes the ICC’s status and powers.• However, Russia had never ratified the statue meaning it was never member subject to its jurisdiction.• Russia was against by ICC’s declaration that Russia’s 2014 annexation of Ukraine’s Crimea peninsula was an armed conflict.Russia is also under international pressure over its campaign of air strikes in Syria over the issue of bombing civilians and civilian targets.• Russia has denied those allegations. Besides, ICC is also examining allegations of war crimes committed by Russian and Georgian forces during a brief 2008 war.Learning: ICC is based in The Hague, Netherlands. It is an intergovernmental organization and international tribunal.It has the jurisdiction to prosecute individuals for the international crimes of genocide, crimes against humanity and war crimes.Q Source: https://www.theguardian.com/world/2016/nov/16/russia- withdraws-signature-from-international-criminal-court-statutehttp://www.thehindu.com/news/national/international-criminal-court- icc-calls-for-bashirs-arrest-delhi-says-no/article7747882.ece80. Domestic freight traffic is crucial for the growth of industrialactivity in India. Highest freight traffic is moved by a) Railways b) Roadways c) Inland Waterways d) Coastal ShippingSolution: b)Learning: Inland waterways in India makes up a paltry 3% of the total transport, compared with China's 47%; in the European Union it stands at 44%. So, C is incorrect. Coastal shipping too has not picked up and constitutes a small share in freight traffic. Maritime Vision 2020 aims at boosting coastal shipping as a freight mover. So, D is also incorrect.Railways accounted for about 42% and roadways for about 58% of India’s total freight traffic in 2011-12. So, B is correct.This is partly linked to an increase in the share of manufactured goods like white goods, fast moving consumer (FMC) goods etc. These cargos move over shorter distances and are time sensitive.Q Source: Page 114: 12th NCERT: India, People and Economy81. Shiqdar officials in Mughal India were mainly responsible fora) Regulating water storage and availability in the provinces b) Maintaining law and order c) Implementing fatwas issued by the emperor d) Supervising work of welfare departmentsSolution: b)Learning: This is an account of important officers appointed in Mughal India.Appointment of two officers with specific responsibilities and jurisdiction was only to avoid concen-tration of authority in one single individual and also to see that one acts as a check over the other.• Sarkars were further subdivided as paraganas.• In each paragana a Shiqdar, an Amin, a Fotehdar, a Munsif, a Hindi writer and a Persian writer were appointed besides the Patwari, Chaudari and the Muqaddam who acted as the intermedi-aries between the government and the people.• The duty of the Shiqdar was to maintain law and order, to collect the revenue and also decide the legal matters.Q Source: Page 214: Themes in Indian History – II: 12th NCERT82. Dual-energy X-ray absorptiometry (DEXA) can be used tomeasure which of these in the human body? 1. Fat content 2. Bone densityWhich of the above is/are correct?a) 1 only b) 2 only c) Both 1 and 2 d) NoneSolution: c)Justification: Statement 1 and 2: Is an enhanced form of x-ray technology that is used to measure bone loss. DXA is today's established standard for measuring bone mineral density (BMD).A DEXA scan is a quick and painless procedure that involves lying on your back on an X-ray table so an area of your body can be scanned. No special preparations are needed before having a DEXA scan.It can also be used to measure total body composition and fat content with a high degree of accuracy.Q Source: Improvisation: UPSC CDS 201383. What are the differences between the Hindustani and

Page 18: stru file · Web view1. Consider the following about Ibn Battuta’s account of India. 1. The highways were safe from robbery and other crimes. 2. Coconut fibre found in India was

Carnatic styles of classical music? 1. Hindustani music uses the concepts of Sruti and Swara which are absent in Carnatic music. 2. Origin of Carnatic music can be traced to Vedas; whereasHindustani music is Medieval in origin.Which of the above is/are correct?a) 1 only b) 2 only c) Both 1 and 2 d) NoneSolution: d)Justification: These are two sub-genres of Indian music.Similarities between both• Similar concepts like sruti, raga, swara, taal• Origin of both can be traced to Vedas (SamaVeda) - The oldest hindu scriptures - and to 13-14th century.• Both monophonic in nature i.e. based around a single melody line, quite elaborate and expressiveHowever, differences exist in nature, style of singing, instruments and technique involved.Differences Hindustani Carnatic• North India • South India• Instruments uses are sitar, • Instruments used are tabla, sarangi, Shehnai etc.violin, veena, gottu,• Chief source is Sangit mridangam Ratnakara of Sarangdeva• Chief source is saint Puranasara Das and carnatic trinity• Emphasis on vocal music: songs are composed to be sung• Number of ragas less• It is not kirti based, it is raga based• Separate repertoire for instrumental and vocal• It has Persian and Islamic influence and is a cultural• Meant to be performed on singing style when played on instruments• Number of ragas is more - 72 ragakarta• It is kirti based while Hindustani raga basedsynthesis of vedic chanting and folk music• Less rhythmic and less • More rhthmically structuredstructured and intensive• Accompanist do not play a than Hindustani major role• Accompanists have much• It can be sung in various larger role styles known as gharanas • It is sung in a single style such as gwalior gharana, jodhpur gharana etc.Q Source: Improvisation: Past year UPSC CSP papers84. ‘Sunway Taihulight’ recently seen in news is a a) Data packet transfer technique using photons b) Genetic engineering method to enhance photosynthetic efficiency of plants c) Sun-bound project of NASA d) World’s fastest supercomputerSolution: d)Learning: China’s ‘Sunway Taihulight’ has won the fastest supercomputer title in the recently released 48th edition of Top 500 list of supercomputers.With this, Chinese supercomputer has retained the top spot in the list of the world’s fastest supercomputers for the eighth consecutive year (rather than USA).Prior to its introduction, Tianhe-2 had claimed top spot in Top500 list for past three years. TaihuLight is intended for use in research and engineering including climate, weather, life sciences, advanced manufacturing and data analytics.Q Source: http://indianexpress.com/article/technology/tech-news- technology/new-chinese-system-sunway-taihulight-named-world- fastest-supercomputer-2865023/85. Some sources of the seventeenth century refer to two terms –khud-kashta and pahi-kashta. They refer to 1. Types of absentee landlordism in rural parganas 2. Land grants given by the Amin to notable individualsWhich of the above is/are correct?a) 1 only b) 2 only c) Both 1 and 2 d) NoneSolution: d)Justification: These terms were used for two kinds of peasants.The khud-kashta were residents of the village in which they held their lands.The pahi-kashta were non-resident cultivators who belonged to some other village, but cultivated lands elsewhere on a contractual basis.People became pahi-kashta either out of choice or out of compulsion. When terms of revenue in a distant village were more favourable peasants moved to other villages. Sometimes they were forced by economic distress after a famine.Q Source: Page 197: Themes in Indian History – II: 12th NCERT86. The USTTAD scheme of the Government of India has abearing on 1. Development of minorities 2. Development of traditional crafts 3. Promotion of Indian arts internationallySelect the correct answer using the codes below.a) 1 only b) 2 and 3 only c) 1 and 2 only d) 1, 2 and 3Solution: d)Justification: Statement 1 and 2: USTTAD stands for Upgrading the Skills & Training in Traditional Arts/Crafts for Development. It was launched by the Union Ministry of Minority Affairs.• The scheme aims at preserving and promoting the rich heritage of the traditional arts & crafts of the Minority communities.

Page 19: stru file · Web view1. Consider the following about Ibn Battuta’s account of India. 1. The highways were safe from robbery and other crimes. 2. Coconut fibre found in India was

• In the light of globalisation & competitive market, these crafts have gradually lost their employability. It also envisages at boosting the skill of craftsmen, weavers and artisans who are already engaged in the traditional ancestral work.Statement 3: The Hunar Haat exhibition was recently organised by the National Minorities Development & Finance Corporation (NMDFC) under “USTTAD” scheme.It seeks to provide an excellent platform to artisans belonging to Minority communities from across nation to display their art and skills before domestic and international visitors.Q Source: http://pib.nic.in/newsite/PrintRelease.aspx?relid=15361987. International Solar Alliance (ISA) is an alliance of solarresource-rich tropical countries lying fully or partially between two tropics. Which of these is/are the objectives of the ISA? 1. Promote standardisation in the use of equipments and processes for generating solar electricity 2. Boost global demands which will result in further reduction in prices of solar energy deployment 3. Adopt mandatory Domestic Content Requirements for solar industries to generate employment and domestic growthSelect the correct answer using the codes below.a) 1 and 2 only b) 2 only c) 1 and 3 only d) 1, 2 and 3Solution: a)Justification: Statement 1: Standardisation will make the manufacturing of equipments and other hardware cheaper.Statement 2: It also envisages boosting R&D, particularly in areas of efficient storage systems.Statement 3: DCRs do not go well with WTO norms, and can be challenged (as India’s Solar Mission DCR was challenged by USA). It is not one of the objectives of the ISA.Learning: More than 120 countries are geographically located in the tropics i.e. between the Tropic of Cancer and the Tropic of Capricorn, either fully or partially. These places get ample sunlight throughout the year, making solar energy easily available resources.These countries also happen to be ones where maximum growth in energy demand in the coming years, but their current production is woefully short of requirement.Thus, the ISA will ensure that as these countries rapidly ramp up their electricity production, predominantly by using clean solar energy and avoid fossil fuels.Q Source: http://economictimes.indiatimes.com/news/economy/policy/internatio nal-solar-alliance-to-be-ratified-at-cop22-in-marrakech-in- november/articleshow/54819284.cms88. Consider the following terms in the context of Medieval Indiaand their correct matches. 1. Jajmani system: Art of craft production 2. Mallahzadas: Cattle traders 3. Milkiyat: Private Land of ZamindarsSelect the correct matches using the codes below.a) 1 only b) 2 and 3 only c) 3 only d) 1 onlySolution: c)Justification: Statement 1: Zamindars in Bengal who remunerated blacksmiths, carpenters, even goldsmiths for their work by paying them “a small daily allowance and diet money”. This later came to be described as the jajmani system, though the term was not in vogue in the sixteenth and seventeenth centuries.Statement 2: Despite the abundance of cultivable land, certain caste groups were assigned menial tasks and thus relegated to poverty.In Muslim communities menials like the halalkhoran, those who cut meat were housed outside the boundaries of the village; similarly the mallahzadas, boatmen in Bihar were comparable to slaves.Statement 3: The zamindars held extensive personal lands termed milkiyat, meaning property. Milkiyat lands were cultivated for the private use of zamindars, often with the help of hired labour. The zamindars could sell or donate these lands at will.Zamindars also derived their power from the state that they could often collect revenue on behalf of the state.Q Source: Pages - Multiple: Chapter 8: Themes in Indian History – II: 12th NCERT89. Geographical Indication (GI) tag is an insignia on productshaving a unique geographical origin and evolution over time. NOT even a single product has been awarded GI status in which of these states in India? 1. Maharashtra 2. Rajasthan 3. Uttarakhand 4. Jammu & KashmirSelect the correct answer using the codes below.a) 1 and 3 only b) 1 and 4 only c) 2 and 3 only d) None of the options (a), (b) or (c) is correct.Solution: d)Justification: Quite recently many products were given GI status in all of these states (other GIs from these states are already there in the list).These are Sangli Raisins (Maharashtra), Parmigiano Reggiano (Italy), Banaras Metal Repouse Craft (Uttar Pradesh), Beed Custard Apple (Maharashtra), Jalna Sweet Orange (Maharashtra), Uttarakhand Tejpat(Uttarakhand), Waigaon Turmeric (Maharashtra), Purandar Fig (Maharashtra), Jalgaon Brinjal (Maharashtra), Solapur Pomegranate (Maharashtra) and Kashmiri Hand Knotted Carpet (Jammu & Kashmir). Learning: The GI registration confers: (i) Legal protection to the products (ii) Prevents unauthorised use of a GI tag products by others (iii) Helps consumers to get quality products of desired traits (iv) Promotes economic prosperity of producers of GI tag goods by enhancing their demand in national and international markets.Geographical Indications are covered as an element of intellectual property rights (IPRs) under the Paris Convention for the Protection of Industrial Property.Q Source: Award of GI status to the above products in India in 201690. Consider the following with reference to Mughal India.1. The Mughal state fixed land revenue on the basis of quality and extent of land. 2. Mughal officials were allowed to collect land revenue in bothcash and kind so that state revenue could be maximized.Which of the above is/are correct?a) 1 only b) 2 only c) Both 1 and 2 d) NoneSolution: c)Learning: The state tried to first acquire specific information about the extent of the agricultural lands in the empire and what these lands produced before fixing the burden of taxes on people. So, 1 is correct.The land revenue arrangements consisted of two stages – first, assessment and then actual collection.

Page 20: stru file · Web view1. Consider the following about Ibn Battuta’s account of India. 1. The highways were safe from robbery and other crimes. 2. Coconut fibre found in India was

Akbar decreed that while he should strive to make cultivators pay in cash, the option of payment in kind was also to be kept open. The officials were instructed accordingly. The state intended to maximize its revenue when fixing and collecting land revenue. So, 2 is correct.Q Source: Page 213: Themes in Indian History – II: 12th NCERT91. Which of these Heads of State or Government of foreign countrieshave NOT been awarded with the Indira Gandhi Prize for Peace, Disarmament and Development? a) Hamid Karzai b) Angela Merkel c) Sheikh Hasina d) Shinzo AbeSolution: d)Learning: You can go through the list. Heads of state or government have been specifically mentioned. https://en.wikipedia.org/wiki/Indira_Gandhi_PrizeIt is accorded annually by India to individuals or organisations in recognition of creative efforts toward promoting international peace, development and a new international economic order; ensuring that scientific discoveries are used for the larger good of humanity, and enlarging the scope of freedom.The prize carries a cash award of 2.5 million Indian rupees and a citation. A written work, in order to be eligible for consideration, should have been published.The recipients are chosen from a pool of national and international nomineesThe first recipient was Parliamentarians for Global Action and then Mikhail Gorbachev.Q Source: Improvisation: UPSC CDS 201392. What are the implications of China being granted “MarketEconomy Status”? 1. India will be bound under WTO rules to negotiate and sign a Free Trade Agreement (FTA) with China. 2. It will severely curb India’s ability to impose anti-dumpingduties on ‘unfairly priced’ Chinese imports.Which of the above is/are correct?a) 1 only b) 2 only c) Both 1 and 2 d) NoneSolution: b)Concept: Under WTO norms, once a country gets MES status, exports from it are to be accepted at the production costs and selling price as the benchmark. Prior to this status, country is considered as a Non Market Economy (NME).Market Economy Status will mean lesser chances of anti-dumping duties being imposed or lesser anti-dumping duties even if they are imposed. This is because a market economy does not exploit the monopoly of the state to unreasonably lower the prices of goods to make them competitive abroad.Justification: Statement 1: There is no such compulsion. WTO operates on multilateral basis. No nation is forced to sign FTAs or treaties with other nations. So, 1 is wrong.Statement 2: The WTO-member countries had decided to deem China as a 'market economy' in anti-dumping cases from December 2016.• As per the 2001 agreement (before this one), to calculate the 'normal value' of exported goods while adjudicating anti-dumping cases, the WTO member nations could ignore the selling price and production costs in China.• India is not inclined to automatically grant the coveted ‘Market Economy Status’ (MES) to China under World Trade Organisation (WTO) norms in December 2016.• The main reason India is reluctant to grant MES to China is that it will severely curb India’s ability to impose anti-dumping duties on “unfairly priced” Chinese imports.Q Source: http://www.thehindu.com/business/india-is-noncommittal- on-market-economy-tag-for- china/article9357908.ece?textsize=small&test=293. Revenue deficit adjusted for interest payment obligations ofthe government is known as a) Primary budget deficit b) Balanced deficit c) Effective Revenue Deficit d) None of the aboveSolution: d)Justification & Learning: Primary deficit is calculated by deducting interest payment from fiscal deficit. So, A is incorrect.Effective Revenue Deficit is the difference between revenue deficit and grants for creation of capital assets. Effective Revenue Deficit signifies that amount of capital receipts that are being used for actual consumption expenditure of the Government. So, C is incorrect.Balanced budget means zero fiscal deficits. So, B is incorrect.Q Source: Improvisation: UPSC CDS 201394. In the Eighteenth Century a number of astronomicalstructures were built by Jai Singh II in west-central India. These are today known by the common name of a) Yantra Vedhshala b) Samarqand Observatory c) Grahadhyaya d) Jantar MantarSolution: d)Learning: Between 1727 and 1734 Jai Singh II built five observatories, all similar, in five different parts of west-central India. All were known by the name Jantar Mantar. They are located at Delhi, Jaipur, Ujjain, Mathura and Varanasi.• He was given the task of revising the calendar and astronomical tablesby Mughal emperor Muhammad Shah.• While the purpose of the Jantar Mantars was astronomy and astrology (Jyotish), they are also a major tourist attraction and a significant monument of the history of astronomy.• The primary purpose of the observatory was to compile astronomical tables, and to predict the times and movements of the sun, moon and planets.Q Source: Improvisation: Chapter 8: Themes in Indian History – II: 12th NCERT95. Consider the following about the Mansabdari System underthe Mughal administration.Assertion (A): The Mansabdari system was organized on hereditary lines.Reason (R): The property of a Mansabdar was not confiscated after his death.In the context of the above, which of these is correct?a) A is correct, and R is an appropriate explanation of A. b) A is correct, but R is not an appropriate explanation of A. c) A is correct, but R is incorrect. d) Both A and R are incorrect.Solution: d)Justification: The Mughal administrative system had at its apex a militarycum-bureaucratic apparatus (mansabdari) which was responsible for looking after the civil and military affairs of the state.• The mansabdari system was not hereditary as appointments were made by the King. Caste system too prevailed in the mansabdari system. So, A is wrong.• Since the property of a mansabdar was confiscated after his death, he used to spend it lavishly during his lifetime. This made the nobles luxurious and it led to their moral degradation which had an adverse effect on their efficiency.

Page 21: stru file · Web view1. Consider the following about Ibn Battuta’s account of India. 1. The highways were safe from robbery and other crimes. 2. Coconut fibre found in India was

Learning: Some mansabdars were paid in cash (naqdi), while the majority of them were paid through assignments of revenue (jagirs) in different regions of the empire.Every civil and military officer was given a ‘mansab’ and different numbers which could be divided by ten were used for ranking officers. It was also meant for fixing the salaries and allowances of officers.Q Source: Page 214: Themes in Indian History – II: 12th NCERT96. Phaldeepika and Brihat Jataka of Ancient India are majorworks on a) Statecraft b) Predictive Astrology c) Healing science d) Vedic chantsSolution: b)Learning: Brihat Jataka is considered as the standard textbook on Vedic astrology, and sometimes described as "India's foremost astrological text”.• It is one of the five principal texts written by Varahamihira, the other four being Panchasiddhantika, Brihat Samhita, Laghu Jataka and Yogayatra.• It is also one of the five major treatises on Hindu Predictive Astrology, the other four being Saravali of Kalyanverma, Sarvartha Chintamani of Venkatesh, Jataka Parijata of Vaidyanatha and Phaladeepika of Mantreswara.• The study of this classic text makes one grasp the fundamentals of astrology.Q Source: Improvisation: UPSC CDS 201397. Consider the following statements.1. He was the Governor General of Bengal when the Permanent Settlement was introduced there in 1793. 2. He defeated Tipu Sultan in third Anglo-Mysore war and signed Treaty of Srirangpatanam. 3. He introduced administrative and police reforms in colonialIndia.4. He declared the practice of Slavery to be illegal.The above refers to?a) Charles Cornwallis b) Cole Mckenzie c) Warren Hastings d) DalhousieSolution: a)Learning: Permanent Settlement: Lord Cornwallis introduced the Permanent Settlement in Bengal, Banaras, Bihar, Carnatic (North Karnataka) and Orissa. As per this system, the Zamindars who formerly collected revenues were recognized as Land Lords and the ownership of the Land was made hereditary. This means that now onwards there would be no auctioning.Police Reforms: Police was under the Zamindars previously. It was taken away from Zamindars and handed over to the Superintendent of the Police at District level. The Police was Europeanized. They were now paid salary and given unlimited powers to arrest the suspected persons.Q Source: Page 259: Themes in Indian History – II: 12th NCERT98. How the Project Progress Monitoring System (PPMS) ishelpful to the Judiciary? 1. It assists judicial administration in reducing the pendency of cases. 2. It ensures greater transparency of information for the litigants. 3. It provides access to legal and judicial databases to the judges.Select the correct answer using the codes below.a) 1 and 2 only b) 1 only c) 1, 2 and 3d) 2 and 3 onlySolution: c)Justification: It is an E-courts mission mode project.• The project implements ICT in Indian judiciary. The project’s scope is to develop, deliver, install and implement automated decision making and decision support system in the courts of India. It also provides regional languages support.• It also makes it possible to provide ICT coverage of judicial process from filing to execution level & also of all administrative activities.• It essentially creates information gateways between courts & public agencies & departments. So, 2 and 3 are correct.• The project is expected to lead to complete demystification of the adjudicatory process thereby ensuring transparency, accountability & cost-effectiveness. So, 1 is correct.Q Source: Improvisation: UPSC CDS 2013 (E-Courts)99. Consider the following about ‘Jogi Arts’ recently seen in news1. It is primarily a tribal art. 2. It is drawn by men only. 3. Its practice is prohibited in nuclear families.Select the correct answer using the codes below.a) 1 only b) 2 and 3 only c) 1 and 3 only d) 1, 2 and 3Solution: a)Justification: The Rajasthan government has installed Jogi Tribal Art Paintings across Jaipur in an attempt to make people aware and to keep the traditional art alive.• The paintings have been installed at one of its stations, on billboards, buses as well as on bus stands around the city.• Jogi Art is tribal art form that hinges on lines and dots. It is mostly done in black and whites; however the recent installation in the city of Jaipur is of vibrant colours.• It is practised by artists from Magriwada in Reodar tehsil of Sirohi district in Rajasthan.• Interestingly, now this tribal art form is just practised by a nuclear family. So, 3 is incorrect. Many women can be seen drawing jogi arts (2 is incorrect).Q Source: http://www.thehindu.com/news/cities/Delhi/exploring- jogi-art/article8207783.ece100. Consider the following statements with reference to theMughal state in Medieval India.Assertion (A): All legal matters were heard and decided only by the Mughal Emperor. Reason (R): All authority in the Mughal state flew from the Emperor.In the context of the above, which of these is correct?a) A is correct, and R is an appropriate explanation of A. b) A is correct, but R is not an appropriate explanation of A. c) A is correct, but R is incorrect. d) A is incorrect, but R is correct.Solution: d)Justification: Many officers in Mughal India handled administration of justice or hearing legal matters. Mughal rulers were the final appealing authorities. For e.g. besides Mughal rulers, Qazi-ul-Quzat was the supreme authority in justice. Since it was difficult for the emperor to do- away with justice in all the cases, he appointed Qazi-ul-Quzat to do the justice in accordance with Muslim Law. So, A is incorrect.Learning: There were other officers as well which handled more than one responsibility.

Page 22: stru file · Web view1. Consider the following about Ibn Battuta’s account of India. 1. The highways were safe from robbery and other crimes. 2. Coconut fibre found in India was

The Amin was in charge of collection of the land revenue and he too tried civil and revenue cases. The Amin and the Shiqdar (who also handled legal hearings) were of the same rank.Q Source: Page 214: Chapter 8: Themes in Indian History – II: 12th NCERT